You are on page 1of 108

Saudi Orthopedic Part One Exam 2008

1. What is the BEST investigation to differentiate between osteomyelitis and bone infarct?
a) Plain X-ray
b) CT
c) MRI
d) Technecium99 bone scan
e) Gallium Scan

2. In anterolateral approach to the hip (Watson-Johnson) the intermuscular plane is


between:
a) Gluteus maximus and rectus femoris
b) Tensor fascia lata and gluteus medius

3. What is the most common organism identified in orthopedic operations?


a) Staphylococcus
b) Streptococcus
c) Pseudomonas

4. What is the earliest finding in compartment syndrome in children?


a) Increase analgesia requirement

5. What can orthopedist do to increase population bone mass?

6. In reflex sympathetic dystrophy RSD:

7. Which substance can antagonize the proteases produced by chondrocyte?


a) Tissue inhibitor of metalloproteinase

8. The following findings can present a pneumothorax EXCEPT:

9. Which of the following cardiovascular disorders is significantly indicating perioperative


complications?
a) Aortic stenosis

10.Patient with distal fibular ankle fracture treated with ORIF with plate complaining of
postoperatively dysesthesia on the top of foot. Motor exam normal. Which of the
following is the most likely cause?
a) Superficial peroneal nerve injury
b) Deep peroneal nerve injury
c) Entrapment of sural nerve with distal end of plate

11.To diagnose ARDS, which of the following lab investigations is appropriate?


a) Plain X-ray
b) ABG
c) CBC
12.Clinically fat embolism differs from air embolism by:
a) Hematuria
b) Breathlessness
c) Interval between injury & symptoms

13.Which of the following statement is the BEST to describe randomized clinical trial study?
a) Demonstrates statistically that one treatment group differs from other treatment
groups
b) Demonstrates statistically that the group exposed to risk factor is different from other
groups

14.What is the most commonly injured nerve in posterior elbow dislocation?


a) Radial nerve
b) Posterior interosseous nerve
c) Ulnar nerve
d) Musculocutaneous nerve

• ulnar nerve injury typically results from stretch


• median nerve injury (rare) typcially associated with brachial artery injury
Saudi Orthopedic Part One Exam 2009
1. 28-year-old man came with fracture shaft of radius associated with complete posterior
interosseous nerve injury. After your examination, you found:
a) Sensory loss of dorsal 1st web. Loss of extension of thumb and fingers
interphalangeal joints. No wrist extension.
b) No sensory loss. Loss of extension of thumb and fingers interphalangeal joints.
Weak wrist extension.
c) No sensory loss. Loss of extension of thumb and fingers interphalangeal joints. No
wrist extension.
d) No sensory loss. Weak extension of thumb and fingers interphalangeal joints. No
wrist extension.
e) Sensory loss of dorsal 1st web. Loss of extension of thumb and fingers
interphalangeal joints. Weak wrist extension.

2. The BEST statement describing osteoinduction:


a) Making frame for new bone to form
b) Revascularization of the graft
c) Stimulates local cells to be bone forming cells

3. 45-year-old man sustained crush injury to his hand in machine with skin loss from the
wrist to the metacarpophalangeal joints. After debridement periosteum of metacarpal
bones was exposed. What is the BEST method for skin cover:
a) Split thickness skin graft
b) Full thickness skin graft
c) Rotation skin flap
d) Local muscle rotation flap
e) Groin flap

4. Normal mineralization seen in which of the following diseases:


a) Reflex sympathetic dystrophy
b) Osteoporosis
c) Renal osteodystrophy

5. In anterolateral approach to the hip (Watson-Jones) the intermuscular plane is


between:
a) Gluteus maximus and rectus femoris
b) Tensor fascia lata and gluteus medius
c) Gluteus maximus and gluteus medius
d) Sartorius and tensor fascia lata
e) Gluteus medius and gluteus minimus

6. In achondroplasia and other chondrodysplasias, the disorder is in:


a) Insulin like growth factor
b) Platelet transforming growth factor
c) Bone morphogenic protein (BMP)
d) Fibroblast growth factor
7. The EARLIST finding in compartment syndrome in children?
a) Increased analgesia requirement
b) Pulselessness
c) Paresthesia
d) Pallor
e) Paralysis

8. Which substance can counteract the protease action of chondrocyte:


a) Zinc
b) Decorin
c) Stromalysin
d) Cathepsin B
e) Tissue inhibitor of metalloproteinases

9. Which of the following cardiovascular disorders is significantly indicating perioperative


complications?
a) Aortic stenosis
b) Myocardial infarction 1 year ago
c) Mitral regurgitation

10. Patient with distal fibular ankle fracture treated with ORIF with plate complaining of
postoperative dysesthesia on the top of dorsum of foot. Decreased sensation to light
touch on the top of dorsum of foot. Motor exam normal. Which of the following is the
most likely cause?
a) Superficial peroneal nerve injury
b) Deep peroneal nerve injury
c) Entrapment of sural nerve with distal end of plate
d) Common peroneal nerve injury
e) Residual effect of compartment syndrome of the lateral leg compartment

11. To diagnose ARDS, which of the following lab investigations is appropriate?


a) Plain X-ray
b) ABG
c) CBC
d) Apical…

12. What is the most commonly injured nerve in posterior elbow dislocation?
a) Median
b) Radial
c) Posterior interosseous
d) Ulnar
e) Musculocutaneous

• ulnar nerve injury typically results from stretch


• median nerve injury (rare) typcially associated with brachial artery injury
13. Immediate source of energy in muscle:
a) Glycolytic
b) Oxidative (Krebs cycle)
c) Adenosine Triphosphate

14. What is the BEST indication of early open reduction and internal fixation?
a) Bimalleolar fracture
b) Carpal scaphoid fracture with displacement 1cm
c) Fracture tibia

15. 12-year-old boy sustained extension type III


a) Closed reduction and immobilization in 30 flexion
b) Closed reduction and fixation with smooth pins
c) Open reduction and lateral column plating

16. What is the amount of blood MUST sustain in 70 kg patient to produce significant
hypotension?
a) 500ml
b) 1000ml
c) 1500ml
d) 2000ml
e) 3000ml

17. Trauma patient came with shock. His blood pressure 60/90 mmHg. You started volume
replacement. What is the BEST indicator for volume status?
a) Urine output
b) Hematocrit
c) Hemoglobin
d) Apical ….
e) Blood pressure

18. Volume resustaion in children:


a) 20ml/kg lactated Ringer
b) 200ml/kg lactated Ringer
c) 20ml/kg normal saline

19. Using adult Spinal board in children you should maintain:


a) Head extended
b) Torso supported with soft layer
c) Head supported with pillow

20. Which of the following statements about end stage osteoarthritis?


a) Decreased water content
b) Decreased proteoglycan
c) Decreased non aggricating proteoglycan
d) Increased number of cells by unit area
21. 35-year-old farmworker sustained a grade II open tibial fracture when he was struck by
a tractor. His last tetanus booster was given when he was16. Appropriate tetanus
prophylaxis for this patient is:
a) 0.5ml of tetanus toxoid
b) 0.5ml of tetanus toxoid and 500units of tetanus human immune globulin
c) 0.5ml of tetanus toxoid and 2.0 million units of penicillin
d) 1.0ml of tetanus toxoid
e) 1.0ml of tetanus toxoid and 500units tetanus human immune globulin

22. Proteoglycan in OA cartilage


a) poor affinity for water but tends to attract each other

23. calcified cartilage


a) 3rd layer of the 4 layers of hyaline cartilage
b) Anchors the cartilage to the underlying subchondral bone

24. When performing a subtalar fusion, inadvertent posteromedial penetration of an


osteotome is most likely to damage the:
a) Tibialis posterior tendon
b) Flexor digitorum longus tendon
c) Flexor hallucis longus tendon
d) Posterior tibial artery
e) Calcaneal (Achilles) tendon

25. osteoporosis risk factors


a) Obesity
b) Mediterranean race
c) Fair skin and hair
d) Delayed menopause

26. Myositis ossificans


a) Characterized by episodes of ossification
b) Deposition of lamellar bone

27. Lisfranc
a) Connects 1st and 2nd metatarsal bases
b) Attached to medial cuneiform to the middle cuniform
c) Attached medial cuneiform to 2nd metatarsal base

28. All risk factors of myositis ossifficans except (Pre-test)


a) Immobilization
b) Thermal injury
c) Physical injury
d) Head injury
e) Paraplegia
29. Muscles supplied by superior gluteal nerve
a) Gluteus maximus, gluteus medius and gluteus minimus
b) Tensor fascia lata, gluteus medius and gluteus minimus
c) Tensor fascia lata, gluteus maximus and gluteus medius
d) Gluteus maximus, gluteus medius and Sartorius

30. bleeding from incision of quadratus femoris:


a) Superior gluteal artery
b) Inferior gluteal artery
c) Medial circumflex femoral artery
d) Lateral circumflex femoral artery
e) Profunda femoris artery

31. Using posterior approach to obtain iliac bone graft you notice that there is bleeding
seems to be from sciatic notch. Which artery is the source of bleeding?
a) Superior gluteal artery
b) Inferior gluteal artery
c) Femoral artery
d) Lateral circumflex femoral artery
e) Medial circumflex femoral artery

32. During an anterolateral (Watson-Jones) approach to the hip, bleeding when the
reflected head of the rectus femoris is detached from the joint capsule most likely
indicates injury to which of the following arteries?
a) Profunda femoris
b) Medial femoral circumflex
c) Ascending branch of the lateral femoral circumflex
d) Transverse branch of the lateral femoral circumflex
e) Common femoral

33. anterior hip smith Peterson internervus plane


a) Femoral nerve and obturator nerve
b) Femoral nerve and superior gluteal nerve
c) Lateral femoral cutaneous nerve and femoral nerve
d) Obturator nerve and inferior gluteal nerve
e) Femoral nerve and pudendal nerve

34. In the anterior approach to the proximal third of the radius, what is the deep muscle
incised to gain access to the bone:
a) Pronator teres
b) Supinator
c) Brachoradialis
d) Biceps
e) Extensor carpi radialis brevis
35. Traumatized patient with fracture forearm. You diagnosed forearm volar compartment
syndrome, which of the following is the most sensitive test?
a) Weakness of metacarpophalangeal joints
b) Discoloration of the hand
c) Pain with passive extension of fingers

36. 18-year-old girl sustained motor vehicle accident had bilateral closed fracture femur
open fracture tibia. 6 hr lucid. Confusion decreased level of consciousness hypoxia
tachypnea. CT brain clear. What is the most likely diagnosis?
a) Pulmonary embolism
b) Fat embolism
c) Epidural hematoma

37. splenic rupture delayed surgery after 2 days altered level of consciousness. What is the
most likely diagnosis?
a) Pulmonary embolism
b) Fat embolism
c) Epidural hematoma

38. closed head trauma post DPL hypothermic # lt femur mid shaft and rt distal 2 butterfly
a) External fixator both femurs
b) Bilateral tibial pin traction
c) Bilateral retrograde intramedullary nailing
d) Plating
e) Antegrade IM nailing

39. bending force


a) Short oblique
b) Transverse with butterfly
c) Spiral
d) Segmental
e) Comminuted

40. Regarding fibrinolytic bathway. The key initial step:


a) Plasminogen to plasmin
b) Fibrinogen to fibrin
c) Prothrombin to thrombin

41. Regarding Virchow’s triad:


a) Venous stasis, endothelial injury, hypercoagulopathy
b) Arterial stasis, endothelial injury, hypercoagulopathy

42. Which of the following muscle is innervated by superficial peroneal nerve?


a) Tibialis anterior
b) Extensor hallucis longus
c) Peroneus longus
d) Peroneus tertius
e) Extensor digitorum longus
43. All the following structures have attachment to the medial femoral condyle EXCEPT
the:
a) Adductor magnus
b) Gastrocnemius, medial head
c) Popliteus
d) Superficial medial collateral ligament
e) Patellofemoral ligament

44. Regarding local anesthetic with epinephrine:


a) Malpractise
b) Good hemostatic effect
c) Good alternative in procedures need cooperative patients
d) Cause necrosis
e) Contraindicated in fingers

45. Blood supply of growth plate


a) Metaphyseal artery
b) Perichondral artery
c) Synovial artery
d) Nutrient artery

46. Medial Scapular winging muscle affected


a) Trapezius
b) Serratus anterior
c) Infraspinatus
d) Supraspinatus
e) Latissimus dorsi

47. # body of scapula associated


a) Vascular injury
b) Rib fractures

48. Amputation in children most common complication:


a) Phantom pain
b) Bone overgrowth
c) Swelling

49. Lady with acute painful swelling medial clavicle for 2 weeks. X-ray, WBC, ESR normal.
What is the appropriate treatment?
a) Irrigation and drainage
b) Excision of medial clavicle
c) Non-steroidal anti-inflammatory agents

50. Health care professional with TB knee MRI


a) Isoniazed, ethamputole and pyridxine
b) Pyrazinamide, pyridoxine
c) Isoniazed, rifampine, pyrazinamide and pyridoxine
d) Isoniazed, rifampine, pyridoxine
51. middle clavicle fracture, nerve injury
a) Radial
b) Ulnar
c) Suprascapular
d) Median
e) Musculcutaneous

52. cell forming alkaline phosphatase responsive to parathyroid


a) Osteoclast
b) Osteoblast
c) Chondroblast

53. The pronator quadratus has the same innervation as which of the following muscles?
a) Flexor digitorum superficialis
b) Flexor digitorum profundus
c) Flexor carpi ulnaris
d) Flexor pollicis longus
e) Palmaris longus

54. Test best for complete tear of rotator cuff


a) Speed test, painful arc,
b) Infraspinatus, drop arm, painful arc
c) Supraspinatus, impingement test, speed test

55. DPL can miss which of the following intra-abdominal injuries:


a) Hepatic fracture
b) Mesenteric laceration
c) Duodenum rupture
d) Jejunum rupture
e) Splenic capsular laceration

56. Polytraumatized patient. Which of the following is the most significant finding?
a) Engorged neck veins
b) tachycardia
c) tachypnea

57. Patient with history of motor vehicle accident. On examination blood pressure 60/40,
hypoxia, trachea shifted to Right and diminished breath sound on the Lt side of chest.
What is the next step?
a) Administration of O2 by mask
b) Chest tube
c) Needle insertion at 2nd intercostal space in mid-clavicular line

58. polytrauma patient with pH= 7.3 pCO2= 40 pO2= 111 lactic acid=?. What is the
appropriate management?
a) Administration of bicarbonate
b) Administration of more intravenous fluid
c) Hyperventilation
59. What is the BEST indicator of good acid base balance in polytraumatized patient after
volume replacement?
a) gastric mucosal pH 7.2
b) lactic acid less than 2.5
c) pH
d) Base deficit - 1

60. 45 years lady Transverse fracture mid-shaft humerus treatment:


a) Hanging cast
b) IM nail
c) ORIF with plate
d) Velpeau dressing
e) Shoulder Spica

61. 15-year-old Child fracture shaft femur Rx


a) Traction
b) Interlocked IM nail from knee
c) Interlocked IM nail from pyriformis fossa
d) ORIF with plate

62. What is the advantage of using full thickness skin graft as compared to split thickness
skin graft to cover palmar skin loss?
a) Less contracture

63. Regarding plate what is the advantage of titanium over stainless steel:
a) High modulus, high yield strength
b) Low modulus, high yield strength

64. Deformation of material as a result of prolonged stress is known as:


a) Plastic deformation
b) Creep
c) Elastic deformation

65. Regarding strain, if material sustained strain beyond its limit is known as:
a) Elastic deformation
b) Plastic deformation

66. After application of above knee cast for fracture tibia you noticed that patient is
complaining of pain and his capillary refill became sluggish. What is the appropriate
next step?
a) Observation
b) Univalving the cast
c) Bivalving the cast
d) Removal of cast
e) Leg elevation
67. To avoid apex anterior angulation in fracture tibial shaft, unilateral external fixation
pins should be directed:
a) Sagittal
b) Coronal
c) Anteromedial midway between sagital and coronal
d) Proximal pins coronal and distal pins saggital
e) Proximal pins saggital and distal pins coronal

68. In diabetic foot ulcer with forefoot amputation what is the most predictive factor for
wound healing?
a) Percutaneous O2
b) ABI 1.1
c) ABI 0.33
d) Albumin level
e) Total protein

69. When peripheral nerve had axonotmesis, the denervated muscle will show signs of
denervation signals on EMG with fibrillation and sharp waves. How long the EMG will
take to be positive after axonotmesis?
a) 5 to 7 days
b) 10 to 14 days
c) 2 to 4 weeks

70. In the ankle joint fibula is transmitting which of the following percentage of body
weight:
a) none
b) 1/6
c) 1/4
d) 1/3
e) 1/2

71. 2 weeks’ pain paresthesia long finger weak wrist flexion holding head on hand
a) C5-C6
a) C6-C7
a) C7-C8

72. Gait studies conducted following a successful, well-positioned ankle fusion have shown
that the loss of ankle motion is compensated for by which of the following
mechanisms?
a) Increased motion in the contralateral ankle
b) Increased motion in the joints of the ipsilateral midfoot
c) An increase in stride length
d) External rotation of the contralateral hip
e) Hyperextension of the ipsilateral knee
73. cells in reaming
a) Has less healing potential due to thermal injury
b) Has less healing potential due to mechanical injury
c) Has same healing potential as iliac bone graft

74. In the posterolateral approach to the radial head, the internervous plane lies between
the:
a) Radial and ulnar nerves
b) Radial and posterior interosseous nerves
c) Posterior interosseous and ulnar nerves
d) Radial and median nerves
e) Ulnar and median nerves

75. The brachial plexus exits the neck between what muscles?
a) Scalenus medius and scalenous posterior
b) Scalenus medius and scalenous anterior
c) Scalenus medius and pectoralis minor
d) Scalenous anterior and scalenous posterior
e) Scalenous anterior and pectoralis minor

76. Using anterior approach to obtain iliac bone graft, what is the neurovascular structure
at risk:
a) Femoral nerve
b) Femoral artery
c) Lateral femoral cutaneous nerve
d) Inferior gluteal artery
e) Superior gluteal artery

77. Suspected cervical spine injury


a) X-ray c-spine
b) Follow ABC

78. Best statement about normal cartilage:


a) Mainly contains type II collagen
b) Avascular aneural

79. If surgeon put 2 different materials together inadvertently, which type of corrosion will
occur:
a) Galvanic

80. In polytraumatized patient with shock (blood pressure=?). In which circumstances


using pneumatic anti-shock garment is contraindicated:
a) Fracture lower limb
b) Traumatic amputation
c) Diaphragmatic rupture
81. In THR when surgeon moved the abductors away from its origin. This well result in:
a) Increased abductor moment arm

82. Menisci EXCEPT


a) Lateral is more mobile than medial
b) Not moving during flexion
c) Mainly type I collagen
Saudi Orthopedic Part One Exam 2012
1. one of the following does not occur with complex regional pain syndrome:
a) Osteopenia
b) Pain in dermatomal distribution

2. what hand infection that most likely lead to osteomyelitis if not treated?
a) Paronychia
b) Web space infection
c) Hypothenar space infection
d) Deep pulp infection

3. What muscle inserted lateral to long head of biceps?


a) Short head of biceps
b) Teres major
c) Pectoralis major
d) Infraspinatus

4. What muscle is responsible for elbow flexion in mid-prone position?


a) Brachialis
b) Biceps
c) Brachioradialis

5. 20-year-old lady with baker cyst in the posterior knee. during surgery, the nerve that
pass medial to popliteal artery and pass between the two heads of gastrocnemius is:
a) tibial nerve
b) common peroneal nerve
c) sural nerve

6. the most severe form of dwarfism is:


a) achondroplasia
b) homozygous achondroplasia
c) hypochondroplasia
d) primordial dwarfism

7. the mode of inheritance in hypophosphatemic rickets is:


a) Mother, son
b) Father, son
c) Mother, daughter
d) Father, daughter

8. the P-value is equal to:


a) 0.05
b) ≤0.05
c) ≥0.05
9. what is the study for rare disease that can be prospective or retrospective?
a) Prevalence study
b) Cohort
c) Case-control

10. branch from the 3rd part of axillary artery:


a) Superior thoracic
b) Lateral thoracic
c) Circumflex artery

11. the radiological study that cannot rule out pulmonary embolism is:
a) normal CT chest
b) Normal CXR
c) normal angiography

12. the structure that doesn’t exit superficial to flexor retinaculum is:
a) ulnar artery
b) radial artery
c) cutaneous branch of median nerve

13. what is the protractor of the scapula?


a) serratus anterior
b) trapezius

14. the site that undergone 2ry ossification at birth is:


a) distal femur
b) distal tibia
c) proximal humerous
d) proximal fibula

15. 20-year-old tennis player presented with acute chest pain after playing, CXR showed 1st
rib #. The responsible muscle is:
a) scalenus anterior
b) scalenus posterior
c) pectoralis minor

16. A patient presented with post op bleeding. All lab values within normal. What is the
cause:
a) inappropriate surgical closure

17. 78,80,83,86,87. Calculate the mean.

18. unilateral triceps reflex weakness is due to:


a) C5 radiculopathy
b) C6 radiculopathy
c) C7 radiculopathy
19. tibial # that will result in 5◦ angulations after fixation with static interlocked IMN is most
likely:
a) Proximal metaphyseal #
b) Transverse diaphyseal #
c) Comminuted #

20. A patient with PH: 7.21, HCO3: 17, Pco2: 12. What is the acid-base imbalance:
a) Metabolic acidosis
b) Metabolic acidosis, resp. alkalosis
c) Metabolic acidosis, resp. acidosis

21. the site of bone subjected to the most tensile strength after bending moment is:
a) Cortex
b) Periosteum
c) Center of bone

22. abduction of the index finger is carried out by:


a) 1st dorsal interosseous
b) 1st volar interosseous

23. a child born with his neck rotated to the left and can not rotate to other side. Dx:
a) Klippel-feil syndrome
b) Congenital muscular torticollis

24. the tibial attachment of PCL:


a) Posterior sulcus. 1.5cm below joint line
b) Posterior sulcus 5mm below joint line

25. 40-year-old with history of old LCL tear presented with medial knee pain. X-ray showed
narrowing joint space and subchondral cysts. Dx is:
a) 1ry OA
b) 2ry OA
c) RA

26.patient s/p old open menisectomy presented with knee pain. The radiographic study for
evaluation is:
a) Non-WB AP
b) WB AP
c) WB lateral
d) WB PA with 45◦ flexion

27.the dampening effect of pivot-shift test is due to:


a) Hip adduction, tibia Internal rotation
b) Hip abduction, tibia internal rotation
28.80 years old patient s/p lumbar laminectomy with repaired dural tear presented with
severe headache and nausea that is severe on standing and disappear on lying flat. Dx is:
a) Orthostatic hypotension
b) Viral meningitis
c) CSF leak

29.while performing epidural anesthesia, the patient suddenly c/o severe back pain
radiating to leg when the catheter was 2cm inside. The action is:
a) Continue epidural anesthesia
b) Withdraw the catheter 1cm
c) Withdraw completely and re-insert

30.in a patient with whiplash injury. What symptom indicates not serious injury?
a) Immediate pain
b) Headache
c) Pain isolated to neck
d) Severe pain for 1 week

31.which knee structure is intra-articular and intrasynovial:


a) PCL
b) Popliteal tendon

32.a patient with meniscus cyst. treatment:


a) Arthroscopy
b) Open cyst excision
c) ITB injection

33.radial inclination normal value is:


a) 22-25
b) 18-22
c) 12-16

34.gunshot injury to leg with tibial #. Vascularity is intact. After debridement, the wound
can be approximated. Dx is:
a) Gutillo II
b) Gustillo IIIA
c) Gustillo IIIB

35.gunshot injury to leg with intact vascularity and small puncture wound. Next step:
a) Perform leg x-ray
b) Operative exploration and debridement

36.the substance that has the lowest modulus of elasticity is:


a) Titanium
b) Cobalt chrome
c) Polyethylene
d) Stainless steel
37.the most common site of Osteochondritis dissecans of the knee is:
a) Lateral part of medial femoral condyle
b) Medial part of medial femoral condyle

38.30 years old lady with right knee pain and swelling, left elbow pain and swelling and right
MPJ pain and swelling. Aspiration of the knee will show:
a) 50000 WBC
b) Low glucose
c) 20% polymorphs
d) Turbid fluid

39.pediatric patient with alkaline burn. ttt:


a) Neutralize with acid
b) Irrigation and neutralization with water

40.the patient with Legg-Calve-Perths disease usually presented with:


a) Painful limp
b) Leg-length discrepancy

41.a patient with anterior sternoclavicular joint dislocation. Trial of closed reduction failed.
Now treatment is:
a) ORIF
b) Repeat closed reduction
c) Conservative treatment

42.with regard to sternal angle. All of the following occur except:


a) Aortic arch bifurcation
b) Tracheal bifurcation
c) Superior vena cava formation
d) 2nd costosternal joint

43.sign of early sepsis:


a) Hypoglycemia
b) Low AV oxygen difference
c) Decrease peripheral vascular resistance

44.which one does not occur in early Erb’s palsy in children?


a) Forearm pronation
b) Elbow flexion
c) Arm adduction

45.which one of the following is not likely due to bone tumor?


a) 4yo child with 1-week history of knee night pain
b) 14 years old boy with 2 days history of knee pain, effusion and hotness
46.with regard to injury to ulnar collateral ligament of the thumb:
a) The proximal end will retract to the aponeurosis
b) The proximal end will entrap in the 1st MPJ
c) Impinge under the adductor polices

47.the tendons harvested for double-loop ACL reconstruction are:


a) Semitendenosus and semimembranosus
b) Semimembranosus and gracillis
c) Gracillis and semitendinosus

48.a child with a fracture involving the physis and epiphysis. This injury is Salter-Harris type:
a) I
b) II
c) III
d) IV

49.one of the following occur in cauda equina compression syndrome:


a) Severe back pain
b) Urinary retention

50.one of the following biochemical markers is of no value in trauma:


a) LDH
b) Ascorbic acid

51.a victim of RTA remain hypotensive after resuscitation with 2 Liters RL. What is the next
step?
a) Cross-matched blood
b) O-ve blood
c) FFP

52.patient with myelomeningocele. All of the following can be associated except:


a) Chiari II formation
b) Mandatory urinary incontinence
c) Paraparesis

53.AO drill bit for gliding hole 4.5mm is


a) 3.2
b) 4.5
c) 6.5

54.patient with treated DVT. The long-term sequel will be:


a) pulmonary embolism
b) Development of stasis ulcer

55.Generalized osteopenia can be seen in all except:


a) Gout
b) multiple myeloma
56.a patient with 4 stab wounds above the nipple who remain hemodynamically unstable
after IVF and blood resuscitation and chest thoracostomy. Next step is:
a) do FAST
b) CT
c) Take him immediately to OR

57.a child with supracondylar humerus # and absent pulses. ttt is:
a) Inform vascular surgeon, closed reduction and fixation in OR then reassess pulses and
if absent or diminished then vascular intervention.

58.hemophilia patient will have spontaneous bleeding when factor VIII is less than:
a) 20%
b) 15%
c) 10%
d) 5%

59.The modality of less value in evaluation for bone tumors is:


a) CT
b) MRI
c) Bone scan
d) Abdominal US

60.patient with brachial plexus injury was found to have weakness in supraspinatus and
infraspinatus. The site of injury is:
a) Upper trunk
b) Upper and middle trunk
c) Lower trunk

61.Patient with lateral compartment syndrome. The affected nerve is:


a) Common peroneal
b) Superficial peroneal
c) Sural

62.Forearm x-ray for a patient s/p both bones fixation. The patient can’t extend her thumb
post op. the nerve injured is:
a) radial
b) posterior interosseous
c) anterior interosseous
d) median

63.AP pelvis x-ray showing dotted ilio-ischial line. This line represents:
a) anterior wall
b) posterior wall
c) anterior column
d) posterior column
64.x-ray showing Galleazi #. The other affected area is:
a) radial head
b) olecranon
c) Distal radioulnar joint

65.Coleman block test is used for:


a) Hindfoot varus
b) forefoot varus
c) midfoot valgus

66.on anterior approach to the hip, the bleeding while you are dissecting is from
a) Ascending branch of lateral femoral circumflex artery
b) Profunda f. art
c) Femoral art
d) descending branch of lateral femoral circumflex artery

67.The intermalleolar distance in 3 YO child is


a) 0-4cm
b) 5-8cm
c) 7-12cm

68.the administration of ciprofloxacin is contraindicated in which of the following patient?


a) Diabetics
b) Alcoholics
c) Renal F.
d) Children

69.which of the structure that provide?


a) ring of LaCroix
b) groove of Ranvier
c) ligaments

70.after the excision of a ganglion cyst in the hand, the structure contains…??

71.flexor tendon sheath of the hand ….?? Anatomical relation

72.multiple trauma Pt with bleeding from mouth and nose with difficulty in breathing and
suprasternal notch retraction what should you do to support breathing?
a) Cricothyroidotomy
b) tracheostomy
c) nasopharyngeal airway

73.from research methodology with given variants they need to calculate RRR (relative risk
reduction)
74.most common cause of death in multiply inj. Pt.:
a) Within 2-3 weeks due to sepsis and multiorgan failure
b) within 1 week with pulmonary embolism

75.Patient with chest trauma, long bone fracture associated with sign and symptoms of fat
embolism the choices of treat.
a) LMW Heparin
b) Warfarin
c) Albumin
d) Filter is not indicated

76.Regarding articular cartilage:


a) Most collagen content is type 2

77.Not in primary survey:


a) maintain ventilation
b) check blood pressure and heart rate
c) cervical spine protection
d) neurological assessment with GCS

78.Treatment for spinal osteomyelitis with S. aures is one of the following except:
a) cefazolin
b) cefuroxime
c) Cipro
d) Tobramycin

79.Content of triangular interval:


a) Radial nerve and profunda brachii

80.How can we diagnose ankle and subtalar stability?


a) physical examination
b) X-ray
c) CT
d) MRI

81.What is the difference between hemorrhagic and cardiogenic shock?


a) high CVP in cardiogenic

82.Case obese boy with limping:


a) SCFE

83.Most common tumor tends to metastasize to the bone:


a) Prostate, thyroid, lung, breast and kidney
b) prostate, thyroid, lung, breast and adrenal glands
c) prostate, thyroid, lung, breast and kidney
d) prostate, parathyroid, lung, breast and kidney
84.All in spinal spondylosis except:
a) increase water

85.baby with femur fracture management:


a) hip spica

86.scenario long bone fracture to decrease lung complications:


a) Immediate fixation

87. hypocalcemia occurs in all except:


a) MM
b) Osteomylecia
c) Rickets

88.question about causalgia and pain in spine

89.regarding muscle, in muscle strength


a) increase mitochondria
b) increase muscle fiber “hypertrophy”

90.Fluid post op
a) standard formalized hospital fluid
b) isotonic fluid with K

91.Patient with long bone fracture, how to decrease lung complication?


a) Early fixation

92.Post MVA, conscious, start to have abnormal chest movement & pigmentation around
his body, what is the diagnosis?
a) Fat embolism

93.After 2 weeks of wound healing, what you will find?


a) Fibroblast

94.When you can use bone graft?


a) Plate fixation with bone defect
b) Both bone forearm fracture
c) Closed femur fracture

95.Regarding blood transfusion


a) Hepatitis B is the most common

96.Regarding foot arches all true except


a) PT support lateral arch
b) Peroneus longus support lateral arch
c) Medial arch composed of medial & middle cuneiform
d) Lateral arch composed of forth & fifth MT
97.10 years old obese with history of fall down complaining of painful limb, on examination
limited external rotation
a) femur fracture
b) SCFE
c) Perthes
d) Septic knee

98.Initiation of the shoulder abduction by


a) Supraspinatus

99.Child with head injury & femur fracture


a) EX fix

100. MVA, unconscious with closed femur & forearm fracture what is the next step
a) Check airway
b) PAN CT
c) OR
Saudi Orthopedic Part One Exam 2013
1. Patient complain of pain after perform sport activity with knee extension and hip flexion:
a) Bicipes femoris
b) Rectus femoris

2. Muscle which have duel innervation:


a) Bicepis brachi
b) FDS
c) FDP
d) Coracobrachialis

3. fat embolism what’s the first laboratory sign?


a) Increase hematocrit
b) high serum fatty acid
c) Fat in the urine
d) hypoxia

4. what's happen in septic shock?


a) Peripheral vasodilation
b) Hypoglycemia
c) Increase lactic acid

5. Patient involve in trauma present with quadriplegia and tachycardia & B/P low Dx is
a) Cardiogenic shock
b) Hypovolemic shock
c) Neurogenic shock

6. which of the following indicates ankle instability in mortise view?


a) Medial clear space 6 mm
b) Talocrural angle 83

7. Calcaneus fracture photo, what is the most causes of postoperative infection?


a) Smoking
b) Type of fracture

• According to the referenced study by Folk et al, the risk of early wound complications is
highest in open injuries, diabetics, and smokers.
• No significant differences were seen in complication rates in terms of: age, sex, other
pre-existing medical conditions, social history, mechanism of injury, time from injury to
surgical stabilization, the type of incision used, use of preoperative antibiotics, or type
of wound closure.

8. what the cell enhancing by PTH to release alkaline phosphatase?


a) Chondrocyte
b) Osteoblast
c) Osteoclast
9. what is the cell present at early of fracture healing?

10.what is the type of collagen present in cartilage?


a) I
b) II
c) VI
d) IX

11.what is the type of open chain should avoid in post OP of ACL?


a) Cycling
b) Leg seated exercise

12. Superficial layer of meniscus is oriented (AAOS)


a) Circumferentially
a) Horizontal
a) Oblique
a) Vertical

13.Regarding ARDS
a) Early fixation decreases the risk of ARDS

14.mechanism of chemotherapy in killing cancer cell


a) Induce apoptosis
b) Direct tumor cell necrosis

15. what muscle cause eccentric contracture during heel strike?


a) Tibialis post.
b) Tibialis Anterior
c) Achilles tendon

16.order of short rotator on lesser sciatic notch


a) Piriformis, superior gemelus, obturator internus, inferior gemelus

17.Early sign of hypocalcemia:


a) Trousseau's Sign
b) Chvostek's Sign
c) Tingling around the mouth
d) Muscle contracture

18.all true regarding hypercalcemia except:


a) Long treatment of calcitonin

19.radiological finding of osteomalacia induce by malignancy Include bone scan and X-rays
finding
a) sunburst appearance
b) cold in bone scan
20.Open fracture and bleeding what you will do first?
a) Dressing and splint
b) I&D in ER
c) Trial of reduction

21.colles fracture you have to know the maneuver of reduction

22.photo for calcaneus of young child complaining of pain on sport injury


a) Apophysitis
b) Avulsions Fx
c) Lytic lesion

23.photo showing the lines of acetabulum identify the posterior column

24.photo of galeazzi fracture and what is the association injury


a) DRUJ

25.indication of pinning of normal head in SCFE


a) Obesity
b) Endocrine disease.

26.gunshot injury with open fracture and wound 2cm no vascular injury and skin are
closeable, Gustello classification
a) I
b) II
c) IIIa
d) IIIb

27.injury through the physis and extend to epiphysis what type of salter-harris
a) III

28.pic. Of growth plate with pointing arrow


a) Resting zone
b) Proliferation
c) Hypertrophic

29.read about axontamesis and EMG finding

30.abduction with overhead activity caused by


a) Serratus anterior & trapezius
b) Infraspinatus & supraspinatus
c) Deltoid & supraspinatus
d) Serratus anterior & deltoid

31.what are the internervous plane in posterior approach of glenohumeral joint?


a) Axillary & suprascapular nerve
32.injury to superficial peroneal nerve cause Weakness on
a) hindfoot eversion
b) Hindfoot inversion
c) Planter flexion
d) Big toe dorsiflexion

33.unilateral triceps reflex weakness is due to:


a) C5 radiculopathy
b) C6 radiculopathy
c) C7 radiculopathy
d) Myelopathy

34.patient with PH: 7.21, HCO3: 11, Pco2: 17, What is the acid-base imbalance:
a) Metabolic acidosis with respiratory alkalosis

35.the radiological study that can’t rule out pulmonary embolism is:
a) CT
b) Angio
c) Echo
d) X-ray

36.patient presented with post op bleeding. All lab values within normal. What is the cause:
a) Vitamin k def.
b) inappropriate surgical closure

37.blood supply of ACL:


a) Middle genicular artery
b) Medial genicular artery

38.40 years old with history of old LCL tear presented with medial knee pain. X-ray showed
narrowing joint space and subchondral cysts. Dx is:
a) Primary OA
b) Secondary OA

39.Talonavicular dislocation post Dx by


a) AP view
b) Canale view
c) Internal oblique
d) External oblique

40.most common Ligament Injured in lateral side of the ankle


a) Calcanofibular Ligament
b) Anterior Talofibular Ligament
c) Posterior talofibular Ligament
d) Anterior tibiofibular Ligament
41.All test positive in OA except:
a) Pivot shat test
b) Lachman test
c) MacMery test
d) Ober's test

42.80 years old patient s/p lumbar laminectomy with repaired dural tear presented with
severe headache and nausea that is severe on standing and disappear on lying flat. Dx is:
a) Orthostatic hypotension
b) Viral meningitis
c) CSF leak

43.the substance that used frequently in orthopedic is:


a) Titanium
b) Polyethylene (UHMWPE)
c) Stainless steel

44.the most common site of Osteochondritis dissecans of the knee is:


a) Lateral part of medial femoral condyle
b) Medial part of medial femoral condyle

45.photo of forearm fracture Rx by ORIF post op pt. cannot extend hid thump what is the
nerve injury
a) AIN
b) PIN
c) Ulnar
d) Radial

46.Question comparing full-thickness skin graft to partial-thickness skin graft.

47.the tendons harvested for double-loop ACL reconstruction are:


a) Semitendenosus and semimembranosus
b) Semimembranosus and gracillis
c) Gracillis and semitendenosus

48.function of posterior spinal Ligament?


a) Tension
b) Support
c) Compression
d) Axis of the body

49.victim of RTA remains hypotensive after resuscitation with 2 Liters RL. What is the next
step?
a) Cross-matched blood
b) O-ve blood
c) Type specific blood
d) Give more fluid
50.patient with treated DVT. The long-term sequel will be:
a) pulmonary embolism
b) development of stasis ulcer

51.stab wound in chest, chest tube shows 1500 ml of blood and pt. now Tachycardia and
hypotensive Rx is
a) Take him direct to OR

52.a hemophilia patient will have spontaneous bleeding when factor VIII is less than:
a) 20%
b) 15%
c) 10%
d) 5%

53.the modality of less value in evaluation for bone tumors is:


a) CT
b) MRI
c) Bone scan
d) Abdominal US

54.Coleman block test is used for:


a) To check flexibility of varus hindfoot

55.anterior approach to the hip, the bleeding while you are dissecting is from:
a) Ascending branch of lateral femoral circumflex artery
b) Profunda f. art
c) Femoral art
d) descending branch of lateral femoral circumflex artery

56.Recurrence Ligament load cause failure of Ligament in:


a) Mid substance (adult)
b) Insertional site (pediatric)

57.Trauma to which bone cause median nerve injury


a) Lunate
b) Scaphoid
c) Traqutrium
d) Trapezoid

58.content of triangular interval:


a) Radial nerve and profunda brachii

59. DDH Rx in 6 months


a) Close reduction, adductor tenotomy, Arthrogram, hip spica
60.One of the following biochemical markers is of no value in trauma:
a) Ascorbic acid
b) Lactate
c) Pyruvate
d) Glucose

61.Adult post MVA, bleeding from nose & mouth, what is the next step:
a) Cricothyroidotomy
b) Nasal intubation
c) ET intubation
d) Trachestomy

62.Fluid Post OP:


a) Isotonic fluid with K
b) Standard formalized hospital fluid

63.most common tumor post. To femur


a) Parosteial Osteosarcoma
b) Periosteal Osteosarcom
c) Periosteal chondroma

64. Anterior Approach to neck cause thoracic duct injury:


a) Left anterior
b) Left lateral
c) Right anterior
d) Right lateral

65.Boundaries of sciatic notch all except:


a) Posterior Sacroiliac Ligament

66.post lateral knee dislocation what is the immediate Rx


a) Angio
b) Doppler
c) Fasciatomy
d) Close reduction

67.to reach to deep leg compartment you have to release which muscle:
a) Soleus muscle

68.MVA patient fast show interabdominal collection and patient still hypotensive after
receiving 2l of LR Rx is:
a) Take patient Now to OR

69.most common knee deformity which keep peroneal Nerve at risk intra OP:
a) Valgus only
b) Valgus and flexion contraction
c) Flexion contraction alone
70.post OP TKR patient c/o unable to do dorsiflexion what you will do
a) Release the dressing and flex knee

71.position which increase hip capsule


a) Abduction, flexion, external rotation

72.60 years old c/o LBP 3/12 back what’s you will do?
a) Reassurance and you will till him this due to mechanical pain
b) Analgesia and PT
c) X-ray, (ESR, CRP, WBC), then upon the result you will proceed for Rx
d) Urgent MRI

73.the goal of high tibial osteotomy:


a) Redistribution of stress in the joint
b) Improve the joint stability

74.what is the difference between hemorrhagic and cardiogenic shock?


a) high CVP in cardiogenic

75.regards wound healing, what you will find


a) fibroblast will appear within 24-36 hrs
b) collagen type I will be predominant at the beginning
c) maximum collagen amount at the first week

76.osteocyte differ from osteoblast because osteocyte


a) have higher nucleus to cytoplasm ratio
b) have a larger number of organelles

77.osteoclast
a) has ruffled border & clear zone

78.the most important part of bone graft


a) osteogenic cell
b) mature osteoblast
c) form scaffold that help to vascularization

79.Complication of pediatric osteomyelitis


a) chronic osteomyelitis
b) growth arrest
c) septic arthritis to the knee

80.13 years old sustain femur fracture what is the fixation method that cause AVN?
a) antegrade IMN with piriformis entry point
b) antegrade IMN with trochanteric entry point
81.72 years old patient with history of fall down complain of Right hip pain & inability to per
weight AP view sow no deformity what is the appropriate radiological study to confirm
diagnosis?
a) CT scan
b) frog lateral x-ray
c) MRI
d) bone scan

82.Patient post op TKR develop numbness over dorsal aspect of the foot, next step:
a) flex knee, remove the dressing bandage

83.maneuver of distal radius fracture:


a) supination, hyperextension
b) pronation, extension, ulnar deviation

84.strongest ligament in hip:


a) iliopubic
b) iliofemoral
c) ischiofemoral

85.in which stage of gait cycle tibialis posterior block the tarsometatarsal joint
a) Toe off

86.x-ray of supracondylar # extension type 3) patient present to ER with elbow swelling,


tether radial pulse, next step:
a) manipulation in the ER
b) close reduction & k wire pining
c) open reduction & k wire fixation

87.pectoralis major rupture commonly involves:


a) clavicle head
b) sternal head

88.which part of article involve the design of study & analytic measure:
a) result
b) discussion
c) abstract

89.best way to do Literature review


a) request help from an expert
b) use internet electronic data base
c) local data base
Saudi Orthopedic Part One Exam 2014
1. regarding polyethylene, the reason of overheating this material is:
a) Enhancing cross-linking
b) Removing free radicals

2. Regarding THR, the least complication that could happen is:


a) Vascular injury
b) DVT
c) Fractures
d) Sciatic nerve injury

3. Best indicator for post op ROM of knee post TKR is:


a) Physiotherapy post op
b) ROM pre-op

4. Different between benign and malignant lesion in X-ray is that benign lesion had:
a) Codman triangle
b) Thinning of cortex
c) Narrow transitional zone

5. which of the following commonly has metastasize to the lung?


a) Giant Cell tumor
b) Osteoid osteoma
c) ABC

6. which of the following has best survival rate?


a) Ewing Sarcoma
b) Periosteal chondrosarcoma
c) Classic osteosarcoma

7. how much collagen present in bone?


a) 95%
b) 90%
c) 85%
d) 80%

8. regarding Clostridium perfrenges


a) Once clostridium found in wound then treatment must be Abx + Surgical intervention

9. regarding Low molecular weight heparin all except:


a) Its action on inhibiting factor Xa.
b) Its metabolized by the liver.

10.which of the following is contraindication for unicondylar arthroplasty?


a) Inflammatory arthritis
b) Young age
11.Regarding multi-drug resistant TB in spine, which of the following antibiotic resistance
happen against?
a) INH + Pyrezinamide
b) Pyrezinamide + Rifampin
c) INH + Rifampin

12.56 years old male complaining of back pain for 3/12 that affect his life and increasing in
intensity, he mentioned history of trauma. MRI was showing lesion involving two
vertebrae, what is the most likely diagnosis?
a) Mets to spine
b) Pyogenic infection of vertebrae.
c) Fracture Spine

13. adult was involved in MVC came with tension pneumothorax on right side and chest
tube was inserted, CXR was taken “was showing tube horizontally at base of the Right
lung”. what to do next? “there was a wide mediastinum in x ray” (Pre-test)
a) Adjust the tube to be vertical.
b) Insert another one in the left side
c) Perform cardeocentesis
d) Perform aortography

14. regarding axontemesis, which of the following is true?


a) Nerve regenerate 1cm proximal to site of injury
d) Nerve regenerate proximal at the site of injury

15.Interval of Anterior Approach to Elbow: (AAOS)


a) BR and Brachialis
b) BR and Biceps Tendon
c) Biceps brachi and Brachialis

16.what indicate Good AP Pelvis?


a) Symphisis pubis would be central to the sacrum.
b) Inferior margin of neck of femur would match with inferior margin of obturator
foramen.

17.regarding Cathepsin K:
a) Bone resorption

18.Enzyme Secreted By osteoclast but not osteoblast:


b) Estrase
c) Acid phosphatase
d) Alkaline phosphatase

19.all of the following causes negative outcome in Acetabular # except:


a) Ipsilateral head of femur fracture
b) Both column #
c) Intra-op complication
20.Early changes of osteonecrosis seen in femoral head:

21.All indication for ORIF Humerus # except:


a) Segmental fracture
b) Vascular injury
c) Shortning > 3cm
d) Radial palsy post closed reduction

22.which of the following seen in cauda equina syndrome?


a) Urinary Retention

23.which is called spring ligament?


a) Calceneocuboid ligament
d) Calceneonavicular ligament
e) Talonavicular ligament

24.Type of joint in Symphysis Pubis:


a) Diarthroidal
b) Amphiarthroidal

25.massive blood transfusion with arrhythmia, Reason:


a) Hypocalcaemia
c) Hypokalemia
d) Hypothermia

26.locked plate can be used in which of the following:


a) Comminuted metaphyseal #

27.Baseball player with pain while catching the ball that he felt his shoulder Slip, he also felt
a dead arm. Most likely diagnosis
a) Gleunohumeral Subluxation
b) Rotator Cuff injury

28.Clear picture of posterior view of pelvic muscle:


a) Quadratus femoris

29.modified judet approach “posterior to scabula”, interval is:


a) Infraspinatus and teres minor

30.posterior cord injury in brachial plexus would affect all except:


b) Latismuss dorsi
c) Subscabularais muscle
d) Teres major
e) Supraspinatus
31.position preferred for patient with basal skull fracture type 1:
a) Simi-sitting
b) Supine
c) Head down
d) Prone

32.which of the following is considered intra-synovial structure of knee?


a) PCL
b) Popletius tendon
c) Semi-membranous

33.best predictor of need for Blood transfusion post op is :


a) Pre-op Hgb
b) Previous history of transfusion

34.tumor that affect both side of joint equally:


a) PVNs
b) Chondrosarcoma

35.Severe osteopetrosis is associated with:


a) Increased osteoblastic activity
b) Aplastic anemia

36.TB spine, which of the following wouldn’t cause paraplegia?


a) Granulation tissue
b) Abscess

37.regarding bone marrow changes from red to yellow with age:


a) It starts from diaphysis to metaphysis
b) Starts in peripheral skeleton to central
c) Starts in central then peripheral

38.Gunshot wound 1cm in tibia comminuted fracture, you want to give tetanus! he didn’t
mention if the patient received immunization before
a) Vaccine + Immunoglobulin
b) Vaccine only
c) No need

39.best view of MRI for ligament injured:


a) T1
b) T2
c) STIR view

40.pre-op things to be done that reduce rate of infection is:


a) Showering with chlorhexidine
b) Shaving
c) Antimicrobial cleaning of surgical wound site
41.child with supracondylar humerus fracture with intact pulses that you decided for O.R.,
upon trials of closed reduction hand became white and pale. what to do next?
a) Explore brachial artery
b) Do bed side arteriogram
c) Extend the limb and splint

42.in medial malleolus fracture, Risk of percutaneous screw is:


a) FHL injury
b) Tibial artery and nerve
c) Tibialis posterior

43.patient came with dislocation of elbow that is simple, after closed reduction you will
immobilize for:
a) 1 week
b) 2 weeks
c) 3 weeks
d) 4 weeks

44.best way to compress Transverse fracture of femur is:


a) Using of plate with concave end “end bend toward the bone”
b) Using a plate with convex end “end bend away from bone”

45.most common cause of allograft failure is:


a) Infection
b) Rejection
c) Non-union

46.best way to assess deltoid ligament in ankle:


a) External rotation stress radiograph
b) Anterior drawer test
c) Talar tilt

47.indication for arthrography of hip all except:


a) Labrum
b) Neck of femur fracture
c) Hip prosthesis

48.most important measure for post op distal radius # :


a) Radial inclination
b) Radial length
c) Rotation

49.in lateral malleolus fracture, using of lateral neutralizing plate has advantage over
antigliding plate in:
a) Less risk for joint screw penetration
b) Less irritation of peroneal nerve
50.in talus fracture, subchondral bone lucency in x-ray 6-week post op would mean:
a) Re-vasculrization
b) Osteonecrosis
c) Arthritis
d) Non-union

51, All has attachment to fibula except:


a) Popletius
b) Soleus

51.7 years old child with distal femur fracture fixed already with resultant varus Right tibia
underwent epiphysiodesis that resulted in 3cm shortening how much further shortening
would be at peak growth if nothing further was done for him
a) < 7cm
b) 7-10
c) 11-14

52.A surgeon recommends an interscalene regional block to a patient undergoing shoulder


arthroscopy. When asked about potential complications, which of the following is most
likely to occur?
a) Persistent motor neuropathy
b) Sensory neuropathy
c) Complex regional pain syndrome
d) Pneumothorax
e) Cardiac arrhythmia & arrest

53.Tibial nerve give branch to all of the following in popliteal fossa except?
a) long head biceps
b) gastrochanemius
c) popletetius

54.All of the following are from superior shoulder suspensory complex except?
a) distal third of clavicle
b) proximal third of clavicle
c) AC ligament
d) CC ligament

55.Gigantism which growth plate affect?


a) Proliferative zone
b) hypertrophy zone

56.Which of the following affect the treatment if lymph node involved?


a) Ewing
b) Angiosarcoma
57.During the storage of the blood (whole blood product) all of the following decreased
except?
a) PH
b) platelet
c) RBC survival time
d) potassium

58.Which nerve supply extensor hallucis brevis?


a) Deep peroneal nerve

59.Which of the following most closely correlates with the peak growth velocity?
a) Olecranon
b) Triradiate cartilage
c) ulnar
d) radius

60.The distance of bone traversed by a screw is defined as which of the following terms?
a) Pitch
b) Working length
c) Screw lead
d) Core diameter

61.All of the following presented with hypercalcemia except?


a) confusion
b) polyuria
c) polydepsia
d) pedal spasm

62.What structure at risk during Thompson approach?


a) PIN

63.CT showing fracture both column what is the best approach?


a) kocher
b) ilioinguinal

64.What is true about vitamin D3?

65.about type of study?

66.about type of bias?

67.Pediatric underwent correction of cubits varus what is most common cause of failure?
a) age at the procedure
b) Fixation failure
c) Rotation
d) type of surgery
68.Which of the following affect the hemostasis?
a) thrompocytopenia
b) factor VIII defect
c) febrinolytic
d) all of the above

69.Forment sign for which nerve?


a) Ulnar
b) median
c) radial

70.Which of the following supply extensor indices?


a) PIN

71.6-year boy fall on his leg, x ray showed incidental finding of a bone lesion on the proximal
tibia. Non-ossifying fibroma? he is walking without pain what is your next step ?
a) CT, MRI
b) Biopsy
c) reassurance and x ray after 3 months
d) CBC, ESR, CRP

72.During anterior approach to obtain iliac crest bone graft, what is the most common cause
to affect the lateral femoral cutaneous nerve?
a) obesity of the patient
b) hematoma
c) Nerve anatomical variation

73.In posterolateral approach to the distal femur. what nerve lateral off sciatic nerve could
be injured?
a) Nerve to short head of biceps
b) nerve to semimembranosus
c) nerve to long head of biceps

74.4 y/o girl with severe head injury and femur fracture. What is the treatment?
a) hip spica
b) hip spica with traction
c) Ex fix
d) ORIF

75.What is the most commonly factor associated with good clinical outcome in neck of
femur fracture?
a) reduction with fracture table
b) accuracy of reduction
c) uses canulated screw
d) uses DHS
76.The most common site of stress fracture?
a) 5th metatarsal
b) proximal tibia

77.30 y/o male FOSH x ray normal. he feels pain in the medial and posterior aspect of the
elbow, What the most commonly injured ligament?
a) Anular ligament
b) LCL
c) Ulnar collateral

78.Fracture Involving the fibula insicura, which direction most commonly unstable for
fibula?
a) Anterior & posterior
b) Medial lateral
c) Rotation

79.Comparing unreamed nail vs reamed nail. The unreamed nail has?


a) Higher in non-union rate
b) Associated with increase pulmonary pressurization
c) Higher in DVT

80.In brachial plexus injury. no elbow flexion which common site of injury?
a) Upper trunk
b) Lower trunk
c) Musculocutaneous

81.All of the following are true about x-ray finding of osteomyelitis except?
a) x ray changes seen before pathological changes

82.Post RTA with lateral CXR. complain of chest pain. what is the diagnosis?
a) Diaphragmatic injury
b) multiple ribs fracture
c) esophageal rupture

83.What is the pathological process in Gaucher's disease?


a) accumulation of glecocerebroside

84.Patient with 20% blood loss, what is the most common finding?
a) normal HR normal BP
b) tachycardia and normal BP
c) tachycardia, normal BP and decreased pulse pressure
d) tachycardia normal BP and normal pulse pressure

85.Titanium compared to stainless steel?


a) Higher tensile strength
86.In hyperparathyroidism?
a) Decrease plasma phosphate level

87.How the metastatic tumor cell causing hypercalcemia?


a) by the tumor invading the bone
b) Mediators act on osteoclast

88.In I.V.F all the following are true except?


a) normal saline contains 155 Na and 155 chloride
b) colloid can be use in hypovolemia

89.What is true regarding Drain?


a) must be removed within 72 hours
b) pennrose associated with infection
c) drain should be through the wound

90.In idiopathic scoliosis the best predictive factor for curve progression is:
a) High peak velocity
b) menarche

91.Increase of lumber foramen in lumber spine by?


a) lumbar extension
b) lumbar flexion
c) lumbar rotation

92.Not used in staph vertebral O.M?


a) Tobramycin
b) Vancomycin
c) cefazolin

93.In blount disease 18 degree?


a) Observation
b) Osteotomy
Saudi Orthopedic Part One Exam 2015
1. In deltopectoral approach guide to deltoid groove is
a) Axillary nerve
b) Musculocutaneous nerve
c) Cephalic vein

2. Pes anserinus innervated by femoral nerve and


a) obturator & sciatic
b) tibial & sciatic
c) saphenous & obturator

3. Superior gluteal nerve innervates


a) gluteus medias, minimus & piriformis
b) gluteus medias, graceless & semitendinosus
c) gluteus medias, minimus & Tensor fasciae latae
d) Gluteus maximus, piriformis & Tensor fasciae latae

4. In proximal volar approach to forearm, plan between


a) brachioradialis & pronator teres
b) brachioradialis & brachialis
c) brachialis & pronator teres

5. which of the following lateral to long head of biceps?


a) Teres major
b) Pectoralis major
c) infraspinatus
d) Latissimus dorsi

6. In posterior shoulder approach which muscle protect radial nerve


a) Teres major
b) teres minor
c) trapezius
d) latissimus dorsi

7. Patient with neck of femur fracture, you decide to fix it what is the Internervous plane?
a) Femoral nerve & obturator nerve
b) Superior gluteal nerve & inferior gluteal nerve
c) Superior gluteal nerve & femoral nerve

8. The tendons harvested for ACL reconstruction are:


a) Semitendenosus and semimembranosus
b) Semimembranosus and gracillis
c) Gracillis and semitendinosus
d) Gracillis and Sartorius
9. A child with a fracture involving the physis and metaphysis. This injury is Salter-Harris
type:
a) I
b) II
c) III
d) IV

10.Early sign of sepsis:


a) Hypoglycemia
b) Low AV oxygen difference
c) Low peripheral vascular resistance
d) respiratory acidosis

11.Patient with gunshot injury to leg with intact vascularity and small puncture wound. Next
step:
a) Perform leg x-ray
b) Operative exploration and debridement

12.80 years old patient post lumbar laminectomy with repaired dural tear presented with
severe headache and nausea, what is the diagnosis
a) Viral meningitis
b) CSF leak
c) Normal post op

13.A patient with PH: 7.21, HCO3: 17, Pco2: 20. What is the acid-base imbalance:
a) Metabolic acidosis
b) Metabolic acidosis, Respiratory alkalosis
c) Metabolic acidosis, Respiratory acidosis
d) Respiratory alkalosis

14.First sign of hypocalcaemia


a) Tingling over the lip & finger
b) Chvostek sign
c) Trousseau sign

15.Which ligament is affected in juvenile tillaux fracture?


a) Anterior inferior tibiofibular ligament
b) calcenofibiular ligament

16.What is the deforming force in bennett fracture?


a) flexor pollicis longus
b) Abductor pollicis longus
c) Extensor pollicis longus

17.What the cell enhancing by PTH to release alkaline phosphatase?


a) Chondrocyte
b) Osteoblast
c) Osteoclast
18.Patient with gunshot injury, open fracture and wound 4 cm, no vascular injury and skin
are closable, what is the gustilo classification?
a) I
b) II
c) IIIa
d) IIIb

19.Patient arrived to hospital after involved in RTA, remain hypotensive after resuscitation
with 2 Litters ringer lactate, what you will give him next?
a) Cross-matched blood
b) O-ve blood
c) Type specific blood
d) Give more fluid

20.Patient with knee dislocation & weak pulse, after reduction pulse become normal what is
the next?
a) Angiography
b) MRI
c) Cast & discharge

21.Patient involved in MVA, FAST done & show interabdominal collection, patient still
hypotensive after receiving 2 litter of ringer lactate, what is the next?
a) Take patient immediately to OR
b) CT scan

22.20 years old lady with baker cyst in the posterior knee. during surgery, the nerve that
pass medial to lateral & above popliteal artery and pass between the two heads of
gastrocnemius is:
a) tibial nerve
b) common peroneal nerve
c) sural nerve

23.Which knee structure is intra-articular and intrasynovial?


a) PCL
b) Popliteal tendon
c) ACL

24.AO drill bit for far hole 4.5mm is


a) 3.2
b) 4.5
c) 6.5

25.A patient with brachial plexus injury was found to have weakness in supraspinatus and
infraspinatus. The site of injury is:
a) Upper trunk
b) Upper and middle trunk
c) Lower trunk
26.45-year-old man sustained crush injury to his hand in machine with skin loss from the
wrist to the metacarpophalangeal joints. After debridement periosteum of metacarpal
bones was exposed. What is the BEST method for skin cover?
a) Split thickness skin graft
b) Full thickness skin graft without fat
c) Full thickness skin graft with fat
d) Groin flab

27.Which of the following cardiovascular disorders is significantly indicating perioperative


complications?
a) Aortic stenosis
b) Myocardial infarction 1 year ago
c) Mitral regurgitation

28.Patient with distal fibular ankle fracture treated with ORIF with plate complaining of
postoperative dysesthesia on the top of dorsum of foot. Decreased sensation to light
touch on the top of dorsum of foot. Motor exam normal. Which of the following is the
most likely cause?
a) Superficial peroneal nerve injury
b) Deep peroneal nerve injury
c) Common peroneal nerve injury
d) Residual effect of compartment syndrome of the lateral leg compartment

29.What is the best indication of open reduction and internal fixation:


a) Bimalleolar fracture
b) Closed tibia & fibiula fracture
c) Closed femur fracture in 5 years’ old

30.35-year-old farmworker sustained a grade II open tibial fracture when he was struck by a
tractor. His last tetanus booster was given when he was 16. Appropriate tetanus
prophylaxis for this patient is:
a) 0.5ml of tetanus toxoid
b) 0.5ml of tetanus toxoid and 500 units of tetanus human immune globulin
c) 0.5ml of tetanus toxoid and 2.0 million units of penicillin
d) 1.0ml of tetanus toxoid
31.When performing a subtalar fusion, inadvertent posteromedial penetration of an
osteotome is most likely to damage the
a) Tibialis posterior tendon
b) Flexor digitorum longus tendon
c) Flexor hallucis longus tendon
d) Posterior tibial artery

32.Multiple trauma patient with severe head trauma, post laparotomy, bilateral femur
fracture
a) External fixator both femurs
b) Bilateral retrograde intramedullary nailing
c) Plating
33.Blood supply of growth plate
a) Metaphyseal artery
b) Perichondral artery
c) Synovial artery
d) Nutrient artery

34.Blood supply to nucleus pulposus receives nutrition via:


a) Diffusion across the vertebral body endplates

35.What is the BEST indicator of inadequate acid base balance in polytraumatized patient
after volume replacement?
a) gastric mucosal pH 7.4
b) lactic acid>2.5
c) Base deficit -1
d) Urineoutof40ml/hr

36.After application of above knee cast for fracture tibia you noticed that patient is
complaining of pain and his capillary refill became sluggish. What is the appropriate next
step?
a) Observation
b) Univalving the cast with serial examination
c) Bivalving the cast
d) Removal of cast

37.To avoid apex anterior angulation in fracture tibial shaft, unilateral external fixation pins
should be directed:
a) far–far in coronal plane
b) far–far in sagittal plane
c) near – far in coronal plane
d) near–far in sagittal plane

38.In diabetic foot ulcer with forefoot amputation what is the most predictive factor for
wound healing?
a) Percutaneous O2 >35
b) ABI > 1.1
c) Albumin > 25

39.When peripheral nerve had axonotmesis, the denervated muscle will show signs of
denervation signals on EMG with fibrillation and sharp waves. How long the EMG will
take to be positive after axonotmesis?
a) 5 to 7 days
b) 10 to 14 days
c) 2 to 4 weeks

40.The best indication of bone scan


a) Acute osteomyelitis
b) Metastasis
41.Fat pad sign with normal x ray:
a) Contusion
b) Occult fracture

42.Proximal humerus responsible for growth


a) 20 %
b) 40 %
c) 60 %
d) 80 %

43. Which of the following tumor arise from epiphysis?


a) Ewing sarcoma
b) Giant cell tumor
c) chondromyxoid

44.Most serious complication in long time post arthroscopy


a) Compartment syndrome
b) Broken instrument

c) Cartilage injury
d) Hemoarthrosis

45.best modality to assess rotator cuff is?


a) MRI

b) Ultrasound
c) CT

d) XRAY

46.Patient with history of dislocated shoulder reduced in ER came to you in the clinic later
on exam full passive range of motion active abduction up to 40 and can’t go more what is
the cause?

a) axillary nerve injury
b) rotator cuff tear

47.what is the cause of biceptal tendinitis?


a) Primary inflammation of the tendon sheath

b) Recurrent subluxation of the tendon in the grove
c) due to acromial impingement anteriorly
d) direct trauma to the shoulder

48.moving the abductors of the hip away from the center of the body will cause?
a) increase abductor moment arm
b) increase body moment arm

c) decrease abductor moment arm
d) decrease body moment arm
49.what will happen in mild to moderate OA?
a) decrease protoglaican content
b) decrease water

50.patient with history of 10 years’ trauma to the right knee came to you with compline of
pain in the knee and morning stiffness for 10 minutes, on exam there is lateral thrust on
walking and opening of the joint on stress test, rom 0- 120 and examination of hand showed
CMC thumb pain, what is the treatment?
a) Constrained total knee replacement
b) Start DEMARDS

51.80 years old lady with history of TKR 3 years back she came with draining sinus from the
knee and on examination you found the extensor mechanism is ruptured what is the
treatment?

a) 2 stage revision arthroplasty
b) 1stage revision arthroplasty
c) Arthrodesis

52.36 years old male status post femoral neck fracture 2 years back came with inability to
walk and x-ray showed non union with 90-degree angle what is the best treatment?

a) valgus producing osteotomy
a) open reduction with locked plate
b) Total hip arthroplasty
c) Vascularized bone graft

53. old patient with tibia deformity and sensory neural hearing loss and high ALP, normal
Ca, phosphate, x ray finding: ivory spine and cotton wool appearance of skull, what is the
diagnosis?

a) Paget disease
b) Fibrous dysplasia

c) Osteosclerotic metastasis

54.pediatric patient with recurrent club foot deformity and have arthrogryposis, what is the
best treatment?

a) Soft tissue release
b) Talectomy
c) mid foot fusion and calcaneal osteotomy
d) talus osteotomy and lateral column shortening osteotomy

55.The most common cause of loose intra-articular body in the knee of young adult?
a) OCD
b) Lose body due to trauma
c) OA
56.hydroxyapatite bone graft action by?
a) osteogenic

b) Osteoinductive
c) Osteoconductive

57.Peak growth velocity best correlate with which physis?


a) radius

b) ulna

c) Olecranon
d) triradiate cartilage
58.diabetic patient came to you 3 weeks after shoulder intra-articular cortisone injection
complain of fever chills on exam erythema and hotness and painful range of motion what is
the treatment?

a) arthrotomy and start parenteral antibiotics
b) aspiration then start parenteral antibiotics
c) oral antibiotic

59.patient post discectomy 2 weeks came with neurologic symptoms and back pain what to
do next?
a) MRI
b) CT

c) Analgesia

60.Pregnant lady post trauma what is the best position in bed?


a) Left lateral
b) Right lateral
trendelenburg with head down
c) trendelenburg with legs down

61.patient involved in RTA hade cervical spine injury you diagnosed dislocation, traction
applied radiological study showed bilateral C6 facet fracture dislocation, what is the best
treatment?

a) halo vest
b) keep traction
c) Posterior spinal fusion from C5-C7
d) Posterior fusion 6-c7 with intra something wires

62.Cubitus varus after supracondylar fracture what is the cause?


a) Rotational malignment
b) Medial column malreduction
c) Lateral column malreduction

63.patient with ankle weber type C fracture after reduction there is syndesmotic instability
what is the most likely injured?
a) Deltoid ligament
b) Anterior tibiofibular ligament rupture
c) calcaneofibular ligament
d) anterior talofibular ligament
64.what is the best view on CT to assess the calcaneal posterior facet?
a) 3D reconstruction

b) sagittal cuts

c) axial semi-coronal
d) parallel to post facet

65.trauma patient stuck under object from below waist he has no pulses distally and cold
limbs and insensate which of the following to be done to improve his outcome?

a) fashiotomy

b) transfer to trauma center
c) CT

66.which one of the following is viewing portal in knee scope?


a) anterolateral

b) anteriomedial

c) parapateral

67.Pediatric patient trauma at hospital after resuscitation with 20ml/kg RL his HR 200
systolic BP 50 what next?

a) 20ml/kg ringer lactate

b) O negative blood
c) Cross matched blood

68.Trauma patient with floating elbow and bilateral femur fracture and other injuries after
stabilizing the patient form life threatening injuries what is the treatment of his humerus
fracture?

a) ORIF
b) Ex fix
c) Nail
d) Cast

69.patient with comminuted posterior column acetabular fracture after ORIF with posterior
approach what to do next?

a) Indomethacin for 6 weeks
b) Radiation after wound healing
c) immobilization
d) Protected weight bearing

70.metabolic response to sever injury?


a) Decrease insulin secretion

b) Decrease aldosterone secretion

c) increase circulating fatty acids
d) catabolism which is reversed after 4 days
71.patient with thoracolumbar burst fracture which of the following is urgent indication for
surgery?
a) loss of bladder function and something about bowl function
b) progressive neurological deficit and radiological evidence of canal compromise
c) pain on palpation the back on log roll

72.adult was involved in MVC came with tension pneumothorax on right side and chest tube
was inserted, CXR was taken “was showing tube horizontally at base of the Rt lung, resolved
pneumothorax & wide mediastinum”. what to do next after tube insertion?
a) Adjust the tube to be vertical
b) Insert an other one in the left side
c) Perform cardeocentesis
d) Perform aortography

73.trauma patient with sever head injury and tibia fracture treated with cast after that the
head injury resolved the patient came later to u on removal of cast patient cant extent his
toes and ankle and decrease sensation of the dorsum what is the cause?
a) Missed compartment syndrome
b) superficial peroneal nerve injury

74.Patient with spine injury brown sequard syndrome which is best to describe it?
a) Ipsilateral motor loses and contralateral lose of pain and temperature
b) Ipsilateral pain and temperature lose contralateral motor lose

75.Patient with C6 quadriplegia he will have function of?


a) Thumb grip
b) Strong wrist flexion function
c) Strong grip strength function
d) Manual wheelchair function

76.57-year-old security he is on vacation for 10 months he wants to do the TKR now because
he wants to go back to work until retirement after 3 years which of the following is the best
indicator that he will go back to work?

a) the patient enthusiasm
b) Undemand work
c) male
d) age

77.patient with subscapularis injury the nerve that supply this muscle comes from which
part of brachial plexuses?

a) posterior cord
b) middle cord
c) lateral cord
d) medial cord
78.patient with femur fracture treated with IM NAIL patient had embolism and died what
should be done at first to prevent this complication?

a) venting the distal femur

b) use deep fluted reamer
c) Use of low pulse lavage

79.what happens in the 5th day of fracture healing?


a) Local PH is low
b) Appearance of chondrocyte at fracture site
c) Increase alkaline phosphatase

80.trauma patient with injury to chest he has minimal decrease of breath sounds on the left
and splinting of chest wall he is hypoxic what is the cause of hypoxia?

a) Flail chest

b) pulmonary contusion
c) hemothorax
d) pneumothorax

81. x-ray of pelvis showing right hip iliac view and left obturator view asking of what u can
see on the right hip?

a) Anterior wall
b) Posterior wall
c) Anterior column
d) Sacroiliac joint

82.patient with multiple hereditary exostosis X-ray showing


malignant mass in the right hip region and asking what is the treatment?

a) Surgical excision
b) Chemo then surgery
c) surgery the radiation

d) Neoadjuvant chemotherapy & excision

83.Old patient known case of metastatic lung cancer she has pain in the thigh the X-ray
showing mass in the femur what you will do next?
a) CT and MRI of femur for staging
b) IM nail of the femur
c) Pain medication and controlling activity

84.Child presented with knee pain, knee scope showed type 1 “complete” discoid meniscus
what is the treatment?
a) complete meniscectomy
b) Arthroscopic reattachment
c) Saucerization
85.female with unknown family history she gives birth for a patient with classic hemophilia
what is the risk of subsequent baby with same disease?

a) 1:2
b) 1:4
c) 1:8
d) 1:16

86.pediatric patient with mass in the thigh muscle biopsy showed neural cells and blue stain
in chromatin, what is the diagnosis?

a) Neuroblastoma
a) Chondrosarcoma
b) Ewing’s sarcoma
c) Osteosarcoma

87.contraindication of unicompartmental arthroplasty?


a) Inflammatory arthritis
b) Medial joint arthritis
c) lateral arthritis

88.80 years old lady had proximal humerus fracture minimally displaced what is the
treatment to have best outcome?

a) functional brace

b) Immobilize and supervised physiotherapy in 2 weeks
c) aggressive physiotherapy
d) ORIF

89.distal tibia fracture after debridement exposed periosteum and tendons what is the best
flap to cover the defect?

a) Free flap
b) Soleus flap
c) gastrocnemius flap

90.The most common cause of hepatitis after blood transfusion?


a) Hepatitis b
b) HIV

91.patient with ectopic lens, macrodactyly, pectus carnetum hypermobility of joints what is
the genetic transmission?
a) Autosomal recessive defect in FBN1 gene on chromosome 15

b) Autosomal dominant defect in FBN1 gene on chromosome 15
c) Autosomal dominant defect in COLA gene

92.Old patient known case of heart disease, involved in RTA he has pain in his chest, on
examination he has congested neck vein & low Blood pressure and tachycardia, what is the
next step?

a) morphine0,1mg/kg
b) norepinephrine
c) give crystalloid fluid
93.The best method to preserve amputated figure?
a) dry cooling with ice
b) emersion in cold normal saline
c) emersion in antibiotics

94.pediatric patient with femur fracture treated by hip spica what is the most common
complication?
a) Leg length discrepancy
b) Angular deformity
c) rotational deformity

95.patient 2 weeks post discectomy came with back pain on examination there is muscle
spasm but no fever, WBC 11, ESR 55 what is the cause?
a) hematoma collection
b) Deep infection

96.patient on warfarin for AS for 8 years undergoing to TKR what is true regarding warfarin?
a) stop warfarin 4 days before surgery then start LMWH HEPARIN and stop the heparin 12
pre op
a) stop warfarin 7 days before surgery then start LMWH HEPARIN and stop the heparin
12 hours pre op
b) stop warfarin 7 days before surgery then start unfractionated HEPARIN and stop the
heparin 4 hours pre op 

c) stop warfarin 5 days only 


97.pediatric patient with fracture of the metaphysis of proximal radius came to ER treated
kept in cast and 2 days in clinic x ray showed angulation 22 degree what is the best
treatment?

a) observe
b) closed reduction

c) percutaneous reduction with pin

98.expansile osteolytic lesion in X-ray is metastasis from which of the following?


a) Kidney

b) Breast

c) Prostate
d) Bronchus

99.best study to view hill sachs lesion of the humeral head is?
a) Internal rotation AP X-ray
b) external rotation AP X-ray
c) scapular AP X-ray
d) lateral X-ray
100.which of the following pass in the greater sciatic foramen above piriformis?
a) Superior Gluteal artery
b) Internal pudandal
c) obturator internus

101.study to be conducted by doctor saying that the medication will cure depression and he
took consent from the depressed patient. what is true regarding his study?

a) the study is correct as he took informed consent
b) study is considered unacceptable as they are vulnerable

102.Calculate epidemiological measure of association


a) 1.5
b) 0.67
Treatment 1 40 60
Treatment 2 60 40
103.pregnant lady came to clinic with husband to check her blood results but the results
were missing from the file the patient was concerned about her blood pressure so you told u
want to take her blood pressure so she extended her arm to you, what is true?
a) Not acceptable u need to sign informed consent
b) extending her arm is considered informed consent

104.Nuremberg code started after


a) after Nazi doctors were convicted of the crimes committed during human experiments
on concentration camp prisoners

105.Study to compare the means of systolic BP in mmhg between smoker and non smoker?
a) T-test
b) Chis quer
c) anova

d) Correlation

106.Recent invention of 4 points regulation for patient autonomy


a) Taking patient consent
b) Do not harm patient

107.which is the mode of 78,87,97,96,79


a) Non
b) 78
c) 97
d) 87

108.what is the mode of distribution of the following numbers 21, 12, 13,13,13,14,14,14, 24
a) Bimodal distribution
b) multimodal distribution
c) unimodal distribution

109.Example for cohort then asked about type of study


Saudi Exceptional Part one exam 2016
1. Example of cohort

2. Example of case - control

3. Example of clinical trial

4. 36, 37, 37, 38, 39, 40, In these values calculate mean, median & mode
a) Mean=37.8, Median=38, Mode=37
b) Mean=37.8, Median=37.5, Mode=37
c) Mean=37.8, Median=38, Mode=38
d) Mean=37.8, Median=37.5, Mode=38

5. HIV & his wife doesn't know & you are treating both of them, the husband asks you to
keep it secret
a) Keep it secret
b) Till the wife
c) Reassure the patient & try to insist him to tell his wife

6. waving & consent

7. girl cutting her wrist due to suicidal


a) Treat her & refer to psychiatrist
b) Refer her to another doctor

8. In smith Peterson approach, Dissection between TFL & Sartorius. Surgeon encounters
bleeding. What the artery damage?
a) Femoral artery
b) Ascending branch of lateral femoral circumflex artery
c) Profonda artery
d) Medial Femoral artery

9. Unilateral triceps reflex weakness is due to:


a) C5 radiculopathy
b) C6 radiculopathy
c) C7 radiculopathy
d) Mylopathy

10. Diabetic patient with PH: 7.21, HCO3: 17, Pco2: 20. What is the acid-base imbalance:
a) Metabolic acidosis
b) Metabolic acidosis, Resp. alkalosis
c) Metabolic acidosis, Resp. acidosis
d) Resp. alkalosis

11. Gunshot injury to leg with intact vascularity and small puncture wound. Next step in
treatment is:
a) Perform leg x-ray
b) Operative exploration and debridement
12. Patient presented to ER with superficial hand wound, had primary closure with
suture. Complaining of bleeding from wound site post closure. All lab values within normal.
What is the cause?
a) Poor surgical closure.
b) Vitamin K deficiency

13. 40 years old with history of old LCL tear treated non-OP, now he is presented with
medial knee pain. X-ray showed narrowing joint space and subchondral cysts. What is the
diagnosis?
a) Primary osteoarthritis
b) secondary Osteoarthritis
c) RA
d) Septic arthritis

14. Gunshot injury to leg with tibia fracture. Vascularity is intact. After debridement, the
wound can be approximated. Which gustillo type?
a) Gutillo II
b) Gustillo IIIA
c) Gustillo IIIB
d) Gustillo IIIC

15. The most common site of Osteochondritis dissecans of the knee is:
a) Lateral part of medial femoral condyle
b) Medial part of medial femoral condyle
c) Medial part of lateral femoral condyle
d) Lateral part of lateral femoral condyle

16. Sign of early sepsis:


a) Hypoglycemia
b) Low AV oxygen difference
c) peripheral vascular dilatation

17. question comparing full-thickness skin graft to partial-thickness skin graft, partial has all
of the following except:
a) Less pigmentation

18. 6 years old boy fall on his leg; he is walking without pain what is your next step?
a) CT
b) MRI
c) biopsy
d) reassurance and x ray after 3 months
e) CBC, ESR, CR

19. most common cause of allograft failure is:


a) Infection
b) Rejection
c) Non-union
20. 30 years old male fall on outstretched hand, x ray normal. he feels pain in the medial
and posterior aspect of the elbow, What the most commonly injured ligament?
a) Anular ligament
b) LCL
c) Ulnar collateral

21. The tendons harvested for double-loop ACL reconstruction are:


a) Semitendenosus and semimembranosus
b) Semimembranosus and gracillis
c) Gracillis and semitendinosus
d) Gracillis and sartorius

22. child with a fracture involving the physis and epiphysis. This injury is Salter-Harris type:
a) I
b) II
c) III
d) IV

23. victim of RTA, patient still hypotensive after resuscitation with 2 Liters RL in the scene.
What is the next step?
a) Cross-matched blood
b) O -ve blood
c) FFP
d) 2 L RL

24. What is the x ray finding in Tumor induced osteomalacia?


a) sunburst appearance
b) coarsened trabeculae
c) sclerosis
• Patients with Tumor-induced osteomalacia display radiographic features of
osteomalacia including generalized osteopenia, pseudofractures, and coarsened
trabeculae. Technecium Tc 99 bone scintigraphy demonstrates diffuse skeletal uptake,
referred to as a “superscan,” and focal uptake at sites of fractures. In general, plain films
demonstrate features of osteomalacia; however, it is impossible to distinguish the
underlying etiology of the osteomalacia with these modes

25. patient with 4 stab wounds above the nipple who remain hemodynamically unstable
after IVF and blood resuscitation and chest thoracostomy 1500cc of blood coming through
chest tube, Next step is:
a) do FAST
b) CT
c) Take him immediately to OR
d) Angio
26. child with supracondylar hummers fracture and thready pulse, what is the initial step in
management?
a) IV pain killer
b) manipulation & splinting
c) closed reduction & pinning
d) open reduction & pinning

27. hemophilia patient will have spontaneous bleeding when factor VIII is less than:
a) 20%
b) 15%
c) 10%
d) 5%

28. The modality of less value in evaluation for bone tumors is:
a) CT
b) MRI
c) Bone scan
d) Abdominal US

29. For a patient post op both bones fixation. The patient can’t extend her thumb which
nerve injured?
a) Radial nerve.
b) Posterior interosseous nerve.
c) Anterior interosseous nerve.
d) Median nerve.

30.The blue line represents:


a) Anterior wall
b) Posterior wall
c) Anterior column
d) Posterior column

31. what is the other affected area?


a) Radial head
b) Olecranon
c) Distal radioulnar joint
d) CMC

32. Adult post MVA, bleeding from nose & mouth, what is the next step:
a) Cricothyroidotomy
b) Nasal intubation
c) ET intubation
d) Trachestomy
33. Triangular interval contents
a) Axillary nerve and humeral circumflex artery
b) Radial nerve and axillary nerve
c) humeral circumflex artery and profunda brachii artery
d) Radial nerve and profunda brachii artery

34. Fat embolism prophylaxis:


a) Low heparin injection
b) Steroid injection
c) early fixation of fracture

35. Fluid Post OP:


a) Isotonic fluid with K
b) Standard formalized hospital fluid

36. What is the different between cardiogenic Shock & hemorrhagic shock:
a) Myocardial Shock present with dyspnea
b) High CVP in cardiogenic

37. Lateral coleman test for:


a) Hindfoot flexibility
b) Forefoot adduction
c) Midfoot rigidity

38. regarding cause of death in multiply injured Patient.


a) Within 2-3 weeks due to sepsis and multi-organ failure

39. muscle injury in knee extension and hip flexion


a) Bicipes femoris
b) Rectus femoris

40. Blood supply of long head of biceps femoris


a) Tibial division of sciatic nerve
b) peroneal division of sciatic nerve

41. muscle which have duel innervation


a) FDS
b) FDP
c) Coracobrachialis

42. Patient involved in trauma present with quadriplegia and Blood pressure is low and
tachycardia Dx is
a) Cardiogenic shock
b) Hypovolemic shock
c) Neurogenic shock
43. fat embolism what’s the first laboratory Sign?
a) Increase hematocrit
b) Lipiduria
• Major criteria
1) Axillary or subconjunctival petechiae
2) Hypoxaemia PaO2 <60 mm Hg; F I O2 = 0.4)
3) Central nervous system depression disproportionate to hypoxaemia
4) Pulmonary oedema
• Minor criteria
1) Tachycardia <110 bpm
2) Pyrexia <38.5°C
3) Fat present in urine
4) Emboli present in the retina on fundoscopy
5) A sudden inexplicable drop in haematocrit or platelet values
6) Increasing ESR
7) Fat globules present in the sputum

44. which of the following indicates ankle instability in mortise view?


a) Medial clear space 6 mm
b) Talocrural angle 83

45. 46. what is the type of collagen present in cartilage?


a) I
b) II
c) VI
d) IX

46. in this type of fracture what is the most causes of


postoperative infection?
a) Smoking
b) Type of fracture

47. what is the type of open chain should avoid in post OP of ACL
a) Cycling
b) Leg seated exercise

48. superficial layer of meniscus is oriented (AAOS)


a) Circumferential
b) Horizontal
c) Oblique
d) Radial

49. mechanism of chemotherapy in killing cancer cell


a) Induce apoptosis
b) Direct tumor cell necrosis
50. In “A”, what muscle cause eccentric contracture?

A
a) Tibialis posterior
b) Tibialis anterior
c) chilles tendon

51. in which stage of gait cycle tibialis posterior block the tarsometatarsal joint?

A B C D E
a) A
b) B
c) C
d) D
e) E

52. What is the early sign of hypocalcemia?


a) Trousseau's Sign
b) Chvostek's Sign
c) Tingling around the mouth & tip of fingers
d) Muscle contracture

53. all true regarding hypercalciemia except


a) Long treatment of calcitonin
b) hypercalcemia mimic hyperglycemia picture
c) vigorous hydration can lead to Ca loss in urine
• The efficacy of calcitonin is limited to the first 48 hours, even with repeated doses,
indicating the development of tachyphylaxis, perhaps due to receptor down regulation

54. feature of osteocyte


a) has higher nucleus to cytoplasm ratio
55. What is the pathological process in Gaucher's disease?
a) accumulation of fat cell
b) accumulation of glycosylceramides

56. Open fracture and bleeding what you will do first?


a) Dressing and splint
b) I&D in ER
c) Trial of reduction

57. indication of pinning of normal head in SCFE


a) Obesity
b) Endocrine disease

58. Provocative test for lateral epicondylitis:


a) Elbow flexion, forearm pronation, wrist flexion
b) Elbow extension, forearm pronation, wrist flexion
c) Elbow flexion, forearm supination, wrist extension

• Greatest tension is elicited with the elbow in extension, forearm in pronation, and wrist in
flexion

59. The arrow represent


a) Resting zone
b) Proliferation
c) Hypertrophic

60. Axontamesis and EMG finding


a)

61. abduction with overhead activity caused by


a) Serritus ant. & trabizius
b) Infraspinatous & superaspinatous
c) Deltoid & superaspinatous
d) Serritus ant & deltoid

62. what are the internervous plane in posterior approach of glenohumeral joint
a) Axillary & suprascapular nerve

63. injury to superficial peroneal nerve cause Weakness on


b) hindfoot eversion
c) Hindfoot inversion
d) Planter flextion
e) Big toe dorso flextion

64. blood supply of ACL:


a) Middle geniculate artery
b) Medial geniculate artery
65. The radiological study that can not rule out pulmonary embolism is:
a) CT
b) Angio
c) Echo
d) X ray

66. Talonavicular orientation best visualized in


a) AP view
b) Canal view
c) Internal oblique
d) External oblique

67. all test positive in osteoarthritis except:


a) Pivot shift test
b) Lachman test
a) MacMery test
b) Ober's test

67. 80 years old patient s/p lumbar laminectomy with repaired dural tear presented with
severe headache and nausea that is severe on standing and disappear on lying flat. Dx is:
a) Orthostatic hypotension
b) Viral meningitis
c) CSF leak

68. the substance that used frequently in orthopedic is:


a) Titanium
b) Polyethylene (UHMWPE)
c) Stainless steel

69. function of posterior spinal Ligament:


a) Tension
b) Support
c) Compression
d) axis of the body

70. Patient with treated DVT. The long-term sequel will be:
a) pulmonary embolism
b) development of stasis ulcer

71. Recurrent load cause failure of ligament in:


a) Mid substance
b) Insertion site

72. DDH Rx in 6 M
a) Close reduction, adductor tenotomy, arthrogram, hip spica
73. Trauma to which bone cause median nerve injury
a) Lunate
b) Scaphoid
c) Traqutrium
d) Tarpizouid

74. One of the following biochemical markers is of no value in trauma:


b) ascorbic acid
c) Lactate
d) Pyruvate

75. most common tumor posterior to femur


a) Parosteial Osteosarcoma
b) Periosteal Osteosarcom
c) Periosteal chondroma

76. ant. approach to neck cause thoracic duct injury:


a) Left anterior
b) Left lateral
c) Right ant.
d) Right lateral

77. boundaries of sciatic notch all except:


a) Posterior Sacroiliac Ligament

78. reach to deep leg compartment you have to release which muscle:
a) Soleus muscle

79. boy with trauma while playing. Ultrasound showed interabdominal collection and
patient still hypotensive after receiving 2 L of LR Rx is:
a) Take patient Now to OR

80. most common knee deformity which keep peroneal Nerve at risk intra OP:
b) Valgus only
c) Valgus and flexion contraction
d) Flexion contraction alone

81. Post OP TKR patient unable to do dorsiflexion what you will do next?
a) Release the dressing and flex knee

82. position which increase hip capsule pressure


a) Adduction, flexion, external rotation
b) Extension, adduction, Internal rotation

83. Feature of osteoclast:


a) Has ruffled border & clear zone
b) Decrease PH & cause bone resorption
84. 60 years old complaining of low back pain for 3 months, progressive & now wake him up
at night what’s you will do?
a) Reassurance and you will till him this due to mechanical pain
b) Analgesia and PT
c) X-ray, (ESR, CRP, WBC), then upon the result you will proceed for treatment
d) Urgent MRI

85. The most important part of bone graft:


a) Osteogenic cell
b) Mature Osteoblast
c) Form scaffold that help to vascularization

86. complication of neonatal tibia osteomyelitis


a) chronic osteomyelitis
b) growth arrest
c) septic arthritis of the knee

87. 72 years old patient with history of fall down complain of Right hip pain & inability to
per weight AP view sow no deformity what is the appropriate radiological study to confirm?
a) CT scan
b) frog lateral x-ray
c) MRI
d) bone scan

88. maneuver of distal radius fracture:


a) Supination, hyperextension
b) Pronation, Extension, Ulnar deviation

89.strongest ligament in hip:


a) Iliopubic
b) iliofemoral
c) schiofemoral

90. common ligament to be injured in ankle sprain:


a) Anterior tibiofibular ligament
b) Calcenofibiular ligament
c) Posterior talofibular ligament
d) Anterior talofibular ligament

91. In distal hummers fracture, what is the complication of operative management:


a) Iatrogenic injury to ulnar nerve
b) Flexion contracture

92. order of short rotator on lesser sciatic notch


a) Piriformis, superior gemelus, obturator internus, inferior gemelus
93. Different between benign and malignant lesion in X-ray is that benign lesion had:
a) Codeman triangle
b) Thinning of cortex
c) Narrow transitional zone

94. regarding Cathepsin K:


a) Bone resorption
Saudi Orthopedic Part One Exam 2016
1. Anterior cord syndrome caused by:
a) Flexion distraction
b) Flexion compression
c) extension compression
d) Extension distraction

2. Victim of three gunshot to the abdomen underwent total small bowel resection and
hemicolocetomy, how much of the small bowel this patient need not to go for parenteral
nutrition?
a) 20 cm
a) 60 cm
a) 100 cm
a) 120 cm

3. best type of enteral feeding for patients going for surgery


a) High calorie
b) High protein
c) Low-residue isotonic formula
d) Elemental

4. A patient on chronic warfarin therapy presents with acute septic knee. INR is 1.4. Which
of the following is the most appropriate management?
a) Proceed immediately with surgery without stopping the warfarin
b) Stop the warfarin, give fresh frozen plasma, and proceed with surgery 

c) Stop the warfarin and proceed with surgery in 8-12 hours
d) Stop the warfarin and proceed with surgery in 24-36 hours

5. case with rhabdomyolysis (urine change to dark color, and acute kidney insult) what's the
best test?
a) haptoglibin
b) creatinine kinase
c) hemoglobin
• serum CK is the most specific test for diagnosis of rhabdomyolysis

6. Ankle fracture, fibula treated with plate and screws, medial malleolus treated with
screws: which of the following is an indication of syndesmotic screw fixation?
a) fracture above syndesmosis
b) Medial clear space widening

7. Nerve injury in posterior approach to forearm


a) PIN

8. Posterior shoulder approach. Vigorous Superior-Medial retraction will injure


a) axillary nerve.
b) suprascapular nerve.
9. A surgeon performs a proximal tibial osteotomy. From the proximal part of the fibular
head, at what level is the peroneal nerve most at risk?
a) 10-30

10.Most common tumor metastasis in bone distal to elbows and knee is from
a) Lung
b) Breast
c) Prostate
d) Other choice

11.case with patient distal radius fracture and numbness reduction done and still has
numbness
a) nerve exploration
b) ORIF with carpal tunnel release

12.Isolated septic neonatal arthritis what s most common organism?


a) Staphylococcus aurous

13.Patient post arthroplasty in ICU. Tachycardia & hypotension and ECG (300 p/min),
irregular irregular rhythm, Rx
a) Pain relives
b) Amiodarone
c) Cardioversion
d) Heparin

14.Patient known case of hemophilia had twisting injury c/o knee swelling
a) Give factor 8 till appropriate level then aspirate
• Joint Aspiration. Arthrocentesis of intra-articular blood may be indicated in acute and
profuse hemarthrosis to prevent long-term joint damage. To be effective, joint
aspiration should be performed within 2 days of bleeding onset, and before the
procedure, factor concentrate must be administered to achieve 100% correction. In
general, however, the application of this procedure should be limited to episodes of
extreme bleeding except for when the hip is involved in a young child where there is a
risk of avascular necrosis.
15.Patient following high tibial osteotomy, what is the most common difficulty in Total knee
replacement?
a) ligamentous laxity.
b) Osteotomy nonunion.
c) Bone loss.
d) Patella baja

16.Osteomalacia that results from liver impairment is characterized by impaired synthesis of


which of the following metabolites?
a) 25-hydroxycholecalciferol
17. 2-year-old boy is seen for evaluation of a limp. His history is significant for a left knee
infection treated with IV antibiotics as a neonate osteomyelitis. Laboratory testing
demonstrates a normal ESR and CRP. The remainder of his workup is negative. An AP
pelvis is seen?
a) Untreated neonatal hip infection
b) Chondrosarcoma
c) Legg-Calve-Perthes disease
d) DDH

18.elderly patient with bone tumor causing articular destruction, what the common cause?
a) Metastasis
b) Osteosarcoma
c) Ewing sarcoma
d) Chondrosarcoma

19.Indication of acute liver transplantation:


a) INR»6.2
b) Age above 40.
c) jaundice if » 3 days of encephalopathy

20.A patient presents to the emergency room with a temperature of 39o C, a heart rate of
115, and a respiratory rate of 25. there are no localizing symptoms and the work-up does
not reveal any specific source for the fever. Which of the following best describes this
patient’s condition?
a) Infection
b) SIRS
c) Sepsis
d) Septic shock

21.Best measure to assess the hip joint subluxation in non-dysplastic hip


a) acetabular index
b) medial joint space widening (waldenstrom Sign)
c) CE angle of wiberg

22.The least common tumor to be malignant; (pre-test)


a) Solitary osteochondroma
b) Solitary enchondroma
c) Multiple osteochondroma
d) Multipe enchondroma

23.Patient undergoing elective surgery and newly diagnosed mild to moderate


hypothyroidism
a) Give thyroxin & then proceed to the surgery
b) Proceed to surgery & consult endocrine after surgery
c) Cancel the surgery & consult endocrine & reach Euthyriod status then do surgery
24.Doxorubicin side effect
a) Cardiac toxicity
b) Encephalopathy
c) Lung fibrosis
d) Myleosupression

25.Skin incision done in wrong site, then surgery continued in correct site, due to
a) Poor communication
b) Poor assessment by resident
c) No critical date at time of surgery
d) Bad training of surgery room staff

26.Neck stab wound in the area b/w origin of SCM and angle of mandible: Rx
a) CT angio
b) Exploration
c) Observation
• If unstable → Exploration
• Zone III if the patient symptomatic → CT angio
• Zone III if the patient a symptomatic → Observation

27.About examining private parts


a) if the same sex
b) doctor and Patient
c) doctor and Patient with chaperon

28.Pediatric patient came several minutes after window injury to thigh with 6cm wound,
bleeding profoundly what to do?
a) Packing with gauze
b) direct pressure
c) tourniquet
d) pressure on the inguinal area or femoral something

29.Male had an MVA, isolated Open distal tibial fracture, neurovascular intact, went for I&D
and had a clean 5*5 cm wound with periosteal stripping, what is the most appropriate
management?
a) soleus flap
b) free flap transfer
c) skin graft

30.Case polytrauma and DPL was done (results given), RBC 20000 μL , WBC 200, what to do
next?
a) Angio
b) Ex fix
c) Laparotomy

• An RBC count of >10,000/μL is considered a positive finding and an indication for


laparotomy; patients with a DPL RBC count between 1000/μL and 10,000/μL should
undergo laparoscopy or thoracoscopy
31.8 years old, 25 kg, femur shaft fracture, Rx:
a) Plate
b) Flexible nail
c) Rigid IM nail

32. 27-year-old man presents to the ED a ER receiving blows to the head. He opens his eyes
with painful stimuli, is confused, and localizes to pain. What is his Glasgow Coma Score?
a) 13
b) 11
c) 9

33.Elderly Patient with DM and ischemic gangrene and rest pain ... pedal pulses absent ...
popliteal felt ... angiography shows tibial artery occlusion, what to do next
a) AKA
b) BKA

34.the highest condensation of mechano receptors


a) Posterior horn of medial meniscus

35.Patient with hypokalemia was 2 after replacement is 3 despite the proper amount, Cause
of refractory is
a) Hypomagnesaemia

36.Pt know case of type II DM, Post TKR, glucose 38, Ketone is negative Ttt :
a) Insulin infusion
b) Aggressive IV fluid
c) Glucose with insulin

37.Maintenance for iv fluid Pediatric 15 kg in 24h


a) 600
b) 1000
c) 1250
d) 1500
38.Maintenance iv fluid for 60 kg adult in 1 hour:
a) 100
b) 110

39.Patient with lupus ... post renal transplant ... post op 2 hrs anuric ... statistical about
donor 10 hrs ischemia and I forgot the rest, what to do next?
a) Angiography
b) CT angiography
c) MRI angiography
d) Doppler U/S
• Accelerated acute rejection, seen within the first few days posttransplant, involves both
cellular and antibody-mediated injury. It is more common when a recipient has been
sensi- tized by previous exposure to antigens present in the donor, resulting in an
immunologic memory response.
40.the appropriate duration of antibiotic therapy for nosocomial urinary tract infection is
a) 3-5 days
b) 7-10 days
c) 21 days
d) Until the patient is asymptomatic and the urinalysis is normal 


41.Which of the following should be given to promote wound healing in patients receiving
steroids?
a) Vitamin A
b) Vitamin B1
c) Vitamin B2
d) Vitamin C

42.Combined GS+Ortho surgery at the same time, which of the following can be used to
decrease the risk of a retained sponge?

a) Limiting the use of sponges by liberal use of suction
b) Routine radiographs in patients undergoing multiple procedures
c) Delaying wound closure until the count is completed
d) Routine radiograph in patients with a BMI >40

43.A 55-year-old man is scheduled to undergo an elective TKR. He has chronic obstructive
pulmonary disease and coronary artery disease, both of which are relatively well
controlled by medication. He would be classified as?
a) ASAClass2
b) ASA Class 3
c) ASA Class 4
d) ASAClass5 


44.During recession of gastrocnemius aponeurosis, what nerve at risk?


a) Lateral peroneal
b) Tibial
c) Common peroneal
d) Medial sural cutaneous

• Sural nerve injury is always a concern with a gastroc recession regardless of whether it
is an open or endoscopic procedure

45.Post Fixation of Talar neck fracture type 2, Walking on the lateral side of the foot, what is
the cause?
a) Varus malunion
b) AVN

46.Posterior knee approach guided by which


a) Medial sural nerve
b) Lateral sural nerve
c) Sphenois vein
47.Post TKR with extension lag 15 degree and flexion 10 degree. What you will do?
a) Revision with proximal taibia component
b) Physiotherapy
c) Manipulation under GA

48.Proximal humerus fracture with minimal displacement for old lady, what is the best for
best functional out come?
a) Immobilization till complete healing of fracture
b) Supervised PT within 2 weeks’ mobilization

49.Child with stable elbow dislocation closed reduction done what will you do next
a) Hinged elbow brace 4weeks
b) Back slab 2 weeks than ROM
c) Immobilization for 1 week

50.What will provide stability in transforaminal sacral fracture?


a) Tension wire fixation
b) Sacroiliac screw in s1 body
c) Triplane osteosyntesis

51.A 73-year-old female presents with persistent right shoulder pain


3 months after undergoing open reduction and internal fixation for
a right proximal humerus fracture. Which of the following could
have best prevented the complication shown in the current
radiograph shown?
a) Insertion of both cortical and locking screws into the humeral head
b) Addition of a 20-gauge intraosseous tension band laterally through the greater
tuberosity
c) Treatment of the fracture with closed reduction and percutaneous k-wire fixation
d) Addition of an inferomedial locking screw within the calcar
e) Intramedullary nailing of the fracture

52.Pipkin 1 fracture after closed reduction CT anatomical reduction what you will do ?
a) Protected weight bearing
b) Skeletal traction with weight bearing after 8wks
c) ORIF

53.Which layer of articular cartilage resistant to shear force?


a) Superficial
b) Middle
c) Deep
d) Tidemark

54.What expected in placing tunnel in 12 o’clock position in ACL recon?


a) Tight flexion
b) Tight extension
c) Rotational instability
55.Which one of the following are factors that will increase risk of patella dislocation after
TKR?
a) External rotation of the tibial component
b) Femoral component more lateral
c) Internal rotation of the femoral component

56.4-year-old girl sustains a buckle fracture of the ulna and associated radial head
dislocation. Closed reduction and immobilization of the arm in 110 degrees of flexion (as
swelling allows) and full supination enhances the stability of the injury by which of the
following?
a) Tightening the interosseous membrane
b) Tightening the biceps
c) Eliminates plastic deformation
d) Relaxes the pronator quadrates
e) Protects the posterior interosseous nerve
• Closed reduction and immobilization of the arm in 110 degrees of flexion (as swelling
allows) and full supination enhances the stability of the injury by tightening the
interosseous membrane and relaxing the biceps tendon

57.Old lady presented to clinic with shoulder stiffness with muscle wasting of shoulder,
what you will do?
a) Physiotherapy stretching
b) Arthroscopic
c) Steroid injection

58.When compared to a patient with OA, a patient with RA who undergoes THA with
cemented components shows inferior results in which of the following categories?
a) functional score
b) femoral component loosening
c) patient satisfaction
d) polyethylene wear

59.Which structure exit though the lesser sciatic?


a) Piriformis
b) Obturator internus
c) Inferior gluteal nerve

60.Post patella dislocation what structure injured?


a) medical facet
b) lateral facet

61.Post total hip arthroplasty with hip pain four month after ruptured diverticulitis with pus
aspiration what you will do?
a) 2 stage surgery
b) Polyethylene change
62.Supracondylar humerus fracture type 3 with cold & absent hand pulse
a) Emergency closed reduction with percutaneous pinning
b) Closed reduction
c) ORIF

63.which of the following will have negative trendelenburg test?


a) Weak Gluteus Maximus
b) Weak Gluteus Medias
c) Coxa vara
d) DDH

• a Trendelenburg sign as evident by the pelvis tilting down on the right during a single-
leg stance on the left lower extremity secondary to weak abductors of the left hip.

64.Following THA, in which of the following is the risk of sciatic nerve injury increased?
a) Adult with neglected DDH

65.Which of the following increase risk of fat embolism and ARDS in patient with long bone
fracture and thoracic injury?
a) Delayed fixation
b) Reamed nailing
c) Unreamed nailing

66.Question asking about type of study, comparing smoking with blood pressure in mmhg, ?
a) T student
b) Chi sq
c) ANOV

67.With fracture distal radius patient is unable to extent his thumb


a) EPL rupture

68.Pt underwent ACL reconstruction 2 years ago is now reporting instability & medial joint
line pain. Examination reveals increased external rotation at 30 degrees of flexion,
negative lachman, negative anterior drawer test, X ray show a mechanical axis of 11
degrees of varus with maintained medial tibial plateau. What's the best next step in
management?
a) High tibial osteotomy with PLC reconstruction
b) High tibial osteotomy
c) TKR

69.One question. About difference of neurogenic and vascular. Claudication

70.brachial plexus injury, with weakness shoulder abduction, winging scapula, location of
injury,
a) upper trunk
b) lower trunk
c) C6 preganglionic injury
71.Posterior sternoclaviclur dislocation with strider congested jugular vein
a) Chest tube
b) Reduction of joint

72.first sensory finding in carpal tunnel syndrome


a) light touch 

b) sharp touch 

c) two-point discrimination 

d) semmes Weinstein monofilaments

73.what the median, 11,12,12,13,13,14,14,15,15,16


a) 13
b) 14

c) 15
d) 16


74.Disc nerve compression with absent ankle reflex, due to


a) L4,5 paracentral 

b) L4,5 paralateral 

c) L5, S1 paracentral
d) L5, S1 paralateral 


75.MVA with spine injury, no neurological symptoms, what’s the indication for surgical
intervention
a) kyphosis of 10 

b) anterior and middle column fracture 

c) posterior column fracture

76.mechanism of injury of chance fracture


a) flexion compression
b) Flexion distraction
c) extension compression 


77.supracondylar fracture, post fixation, injury to AIN, came to clinic 8 weeks after
discharge, no improvement in nerve injury, next
a) nerve conduction study
b) exploration
c) observation

78.supracondylar fracture, symptoms of AIN injury, what muscle supplied by the injured
nerve.
a) pronator teres, lumbricles, FDP 

b) FDP, pronator teres, FPL 

c) FDP, pronator quadratus & FPL
79.anterior cervical approach, patient complaining of voice changing this due to injury of
a) recurrent laryngeal nerve

80.post anterior shoulder dislocation what’s the most common ligament to be affected
a) anterior-inferior glenohumeral ligaments

83. case with acromioclavicular dislocation, what’s the indication of surgical intervention?
a) posterior displacement of clavicle
b) clavicle below coracoid 


84. child at age 6 weeks, best for DDH evaluation


a) US
b) x-ray
c) CT

85. best for prognosis in CP patient


a) ligamentous laxity at 1 y
b) walking at age of 5 to 7 years

86. 3 years old boy minimally displaced proximal metaphysical tibia fracture 6 months prior
to presentation was treated with long leg cast with 12-degree valgus malunion, ttx
a) follow up radiograph in 6 months
b) close wedge proximal osteotomy

87. ankle brachial index result makes you suspect vascular injury,
a) 1
b) 0.9
c) 0.7
d) 0.55

• An ABI <0.9 correlates with increased risk of myocardial infarction and indicates
significant, although per-haps asymptomatic, underlying peripheral vascular disease.
Normal is more than 1.
• Patients with claudication typically have an ABI in the 0.5 to 0.7 range, and those with
rest pain are in the 0.3 to 0.5 range. Those with gangrene have an ABI of <0.3. These
ranges can vary depending on the degree of compressibility of the vessel. The test is less
reliable in patients with heavily calcified vessels. Due to non compressibility, some
patients such as diabetics and those with end-stage renal disease may have an ABI of
1.40 or greater and require additional noninvasive diagnostic testing to evaluate for
peripheral arterial disease (PAD).

88. lateral x ray of foot showing calcaneus fracture with


skin tenting, indication for urgent surgical intervention.
a) tendoachelis contraction
b) skin complications
89. acetabula posterior wall fracture, best in
a) AP pelvis
b) obturator oblique
c) Iliac oblique

90. Postoperatively a 70-kg patient has a serum sodium value of 130 mEq/L. To
increase this value to 140 mEq/L would require the administration of how much
sodium in mEq?
a) 140
b) 250
c) 420
d) 600
e) 740

• Na Deficit = 0.6 X Normal Weight X (Desired Na – Serum Na)

91. A 16-year-old male presents to the emergency department one day after scratching his
leg on a piece of scrap metal. He reports a progressive rash on his leg that has
advanced over the last several hours. In the emergency room his temperature is 102.8
degrees and his systolic blood pressure is 98 mmHg. On physical exam the clinical
finding shown in Figure is found. What would be the most appropriate next step in
treatment?
a) MRI
b) Biopsy with urgent frozen section in the operating room
c) Needle aspiration
d) CT
e) Ultrasound
Saudi Orthopedic Part One Exam 2017

1. 78 years old female sustained fall down in the bathroom with Right hip pain and unable
to walk X-ray was normal.
Which radiological study will be helpful?
a) CT
b) Traction with internal rotation AP
c) Iliac view

d) Obturator

2. 22-year-old sustained proximal tibial fracture, IM nail Planned. Which of the following
deformities is most likely associated with this procedure?
a) Misalignment
b) Anterior apex, valgus
c) Posterior apex, valgas
d) Posterior apex, varus

3. Pediatric distal femur fracture (Lateral Condyle) post screw fixation .. Pt will have?
a) Complete arrest

b) Partial arrest with geno valgus
c) Partial with geno varus

4. RTA with Head injury with bilateral open femur and pelvis fracture, post Laparotomy and
splenectomy and irrigation of the puncture wounds, what will you do next?
a) Bilateral Ex Fix and pelvis Ex Fix
b) Bilateral retrograde nail and pelvis ex fix
c) Bilateral antegrade and pelvis ORIF

5. Which fluid best for irrigation of tibia gastillo type 3 fracture


a) Normal saline
b) Ringer lactate

c) Iodine with ringer lactate

d) Ringer lactate with gentamycine

6. Deforming forces of Subtrochanteric femur fracture


a) Iliopsoas and gluteus medius

7. Patient with femur fracture most common cause of malrtotaion


a) Fracture comminution
b) Close reduction
c) Surgeons experience

8. Comminuted calcaneus fracture 3 months back, presented with clawing of the lesser toes.
What is the most likely cause?

a) Missed compartment syndrome
b) Weakness of the extensor muscles
c) Lateral planter neuropathy
9. Most common Sacral fracture associated with neurological deficit:
a) Sacral iliac fracture and dislocation
b) Fracture lateral to the sacral foramina
c) Fracture medial to the sacral foramina

10. Patient with RTA underwent ORIF of the symphysis pubis and bilateral percutaneous
screw fixation of the sacroiliac joint, what is the common complication?

a) lose of great Toe dorsiflexion

11. Poly trauma patient with Open pelvis, pelvic binder was applied in the ER, Fast -ve , Pt is
till hypotensive and tachycardia. What is the appropriate management?

a) Embolization followed by External fixation

b) External fixator then laparotomy
c) Laparotomy and packing of the pelvis

12. Best fixation for posterior acetabular wall comminuted fracture


a) Cable
b) Buttress plate
c) K-wires

13. 10 y/o with posterolateral fracture dislocation of elbow, following close reduction. What
is an absolute indication of ORIF of the medial condyle?
a) Displacement > 0.5 mm on AP radiography.

b) Entrapment of fragment in the elbow.
c) Displacement 75% on CT scan.

d) If occur in dominant arm of athlete.

14. 11 y/o with Supracondylar humerus fracture, Blauman angle 88


a) Close reduction and k wire
b) open reduction and k wire - ORIF
- Close reduction and cast

15. 7 years old c /o mild pain at forearm since 2 years. X-ray shows radiocapitular anterior
dislocation Management?
a) Physiotherapy if not improved surgical treatment

b) Ulnar osteotomy and radial head close reduction and annular ligament reconstruction
c) Close reduction and casting

d) Annular ligament reconstruction

16. Which of the following have the greatest risk of developing compartment syndrome.
a) 14 years old with Displaced transverse tibia fracture
b) 11 years old with Closed forearm fracture.

c) 4 years old with Supracondylar fracture type 3 with intact radial pulse.
d) 5 years old with displaced supracondylar fracture and ipsilateral non-displaced ulna
and radius fracture
17. 12 years old boy developed a close injury to right ankle distal neurovascular was intact.
X-ray showed a medial malleolus fracture SH type 4. What is the best management?
a) Closed reduction and weekly x-rays. 

b) Closed reduction and percutaneous pinning. 

c) Open reduction and internal fixation + cast.
d) Distal tibia and fibula epiphyseodesis.

18. Patient with distal tibia fracture (Pilon fracture) with lateral malleolus, mild swelling soft
tissue intact. What is the best management?
a) Immediate ORIF of the lateral malleolus and Ex fix tibia
b) Immediate ORIF for both
c) Splinting
d) EX food for both

19. Distal Tibia fracture and proximal fibula fracture what should be done pre-OP 

a) Ankle X-ray
b) knee x-ray
c) Knee CT
d) Ankle MRI

20. Lateral malleolus fracture, to rule out deltoid injury 



a) Stress view

21. Which of the following indicate syndesmotic injury


a) Tibiofibular clear space 6 mm

b) Positive squeeze test

22. X-ray of 10 y old boy basketball player with trauma to the knee, x-ray
showed high patella with avulsion of the tibial tuberosity.
What is associated with this condition?
a) Avulsion of patellar tendon at proximal tibia
b) Growth arrest with Recurvatum deformity
c) Compartment syndrome

23. Pediatric patient with trauma to the elbow, the best view to evaluate for a lateral
condyle fracture

a) AP
b) Lateral
c) Internal Oblique
d) External Oblique

24. 47 years old male history of Gunshot with medial thigh big hematoma and deformity
before 3 hours, weak pulses distal to the injury and decrease sensation, x-ray mid shaft
femur fracture, what to do next?
a) Angiography
b) External fixator application with fasciotomy
c) Open repair of the artery - Intramedullary nail
25. Distal radius fracture with decrees sensation, underwent ORIF with volar plate .. post OP
worsening of the numbness over the thumb and index finger?

a) Urgent carpal tunnel release
b) Fasciotomy
c) Remove the hardware
d) Arm elevation

26. Distal humerus fracture type C3 post ORIF, what is the most common complication?
a) Decrease elbow ROM

27. Tibial plateau fracture schatzkar 5 treated with double plating, the most predictive of
post OP arthritis
a) Alteration of Mechanical access
b) Pre OP fracture depression

c) Fracture type

d) Age

28. 12 years old boy football player C/O of left knee pain for the last 3 months, no hx of fall
or twisting injury O/E mild swelling, hotness or effusion on physical exam DX :

a) ACL

b) Medial meniscus
c) Osgood Schlatter
d) patellar tendonitis

29. Picture with Press Belly test


a) Subscapularis

30. Posterior cruciate insufficiency diagnosed using the quadriceps active test is confirmed
with tibial translation
a) anteriorly at 20 to 30 degrees of flexion.
b) anteriorly at 70 to 90 degrees of flexion.
c) posteriorly at 20 to 30 degrees of flexion.
d) posteriorly at 70 to 90 degrees of flexion.

31. A 25-year-old male presents with a 2-month history of anterior shoulder pain. He is
noted to have normal forward flexion and abduction strength and isolated weakness on
shoulder external rotation. o/e there is infraspinatus muscle wasting. He has no numbness
or paresthesia. MRI as shown ..
a) Brachial plexus injury
b) Spinoglenoid notch cyst and labral tear
c) Axillary nerve palsy
d) Suprascapular notch cyst

32. 12 years old boy complaining of knee pain and effusion related to activity
(AP knee x-ray provided), which of the following considered prognostic for good outcome?
a) Weight reduction
b) open growth physis
33. During an ACL Reconstruction which of the following is the most accurate position to
avoid impingement when placing the graft at the tibial side
a) center footprint
b) anterior footprint
c) posterior footprint
d) posterior footprint and 7mm anterior to PCL

34. 70-year-old male with chronic shoulder dislocation, forward flexion from 0 - 80.
Complaining of mild shoulder pain. Treatment consist of:
a) Hemiarthroplasty
b) Conservative therapy & pain management
c) Open reduction & subscupularis advancement - Close reduction & sling

35. 23-year-old male with RT knee twisting injury c/o pain and swelling with full ROM A
"double PCL sign" seen on a sagittal MRI image of a knee is indicative of which of the
following conditions?
a) ACL tear.
b) Combined ACL & PCL tears.
c) PCL & PLC tears.
d) Bucket-handle meniscal tear

36. A 22-year-old swimmer has pain and weakness in her dominant shoulder with overhead
use. She feels that her shoulder goes in and out. What is the most appropriate surgical
management?
a) Inferior Capsular shift
b) Bankart repair
c) Latarjet procedure

37.which is the most common site of compression of the Suprascapular nerve?


a) Suprascapular notch
b) Spinoglenoid notch

38. 55 y/o Male patient, Right hand dominant c/o weakness of Right shoulder after
reduction of Anterior Shoulder dislocation. What is the most common cause?
a) Rotator cuff tear

39. Which of the following is static stabilization of posterior lateral corner?


a) Lateral gastrocnemius head
b) Popliteofibular ligament
c) Tensor fascia lata

40. Mechanism of cervical injury and Quadreparesis in football player:


a) Flextion
b) Rotation
c) Hyperflextion
d) Hyperextention
e) Axial load
41. During total uncemented hip replacement patient developed hypotension and hypoxia,
the cause?

a) Fat embolism
b) Allergic reaction to anesthesia medication

42.Contraindication to Unicondylar knee replacement


a) Inflammatory arthritis

43. The most common early complication post THR?


a) DVT
b) Dislocation


44. Predictive of post OP ROM after TKA


a) Pre-OP ROM

45. What is the cause of Patellar malt racking post TKR


a) Medialization of femoral component
b) External rotation of femoral component
c) 5 degrees external rotation of tibial component
d) Medialization of the patella

46. Obese 10 years old boy c/o of right knee pain and limping, no history of fever or trauma
on exam the leg goes on external rotation when flexing the hip Dx :

a) AVN

b) SCFE
c) iliotibial band syndrome
d) osgood schlatter

47. 10-year-old girl c/o abnormal gait and falling down for the last 9 months. Examination
showed bilateral genovalgum.
What is the most accurate sentence describing the
deformity?
a) Sever regressive symmetric.
b) Mild progressive symmetric.
c) Sever progressive localized.
d) Sever symmetric generalized.

48. In relation to rotational growth of lower extremity, which of the following statements is
correct:

a) In Toeing usually spontaneously corrected
b) Most of deformities require surgical correction
c) Both femur and tibia tend to medially rotate d. 20% Will end up with functional
disability

49. DDH in US



a) Alpha angle>60, Beta<40
b) Alpha<40, Beta>60
50. 4 months child has asymmetrical gluteal folds. What is the most accurate statement?
a) It is most common in boys with breech position

b) Open reduction

c) Galeazi sign when knees unequal after knee and hip flexion
d) Ortolani flex hips 90 degrees and try to dislocate the joint

51. Mylomeningiocele at the level of L4 admitted for surgical intervention with no known
allergy

a) Latex skin allergy test

b) Observation latex free protocol
c) Regional better than GA

52.In pediatric orthopedic which one is true?


a) SCFE may present with knee pain
b) Perthes is AVN of the humeral head 

c) 90% of SCFE is bilateral 


53. 10 months old DDH with bilateral pure acetabular dysplasia (AI 38) :
a) Pavlic harness
b) Static abduction splint
c) Close reduction and SPICA cast
d) Open reduction with pelvic osteotomy

54. Regarding Becker muscular dystrophy is a deficiency in


a) Dystrophin

b) Protien

c) Myosin

55. About Distraction Osteogenesis, which is true?


a) 2 mm per day

b) Use cartilage model

c) Maximum latency phase 10-14 days
d) High oxygen tension

56. Which one of the following if operated would have good outcome?
a) 14-year-old with severe genovalgus deformity
b) A patient with genovalgum deformity and has 2 years of growth remaining

57. 12 year old with complaining of pain upon forward bending .. X-ray showed (( left
thoracolumbar curve )) .. What is the next step in management?

a) Mylogram

b) CT
c) xray AP and lateral

• A similar Q was in the previous years common questions, but they mentioned that the
pain was improved by NSAID .. So the answer is CT as the Dx is Osteoid osteoma “”
58.Spinal cord injury with upper more than lower
a) Central cord syndrome
b) Anterior Cord syndrome

59. What is the appropriate pairing



a) C4 Electric wheel chair mobilization with mouth
b) C6 Manual wheel chair
c) C7 Independent

60. Post ACDF 3 hr post extubation patient was c/o stridor and SOB .. what is the next step
in management?
a) Evacuation of hematoma
b) Maintain airway
c) CT scan

61.Pt post discectomy 2 years back c/o of pain DD recurrence or fibrosis the study of choice
a) MRI with gadolinium

62. Patient with spine injury T10 what indicate good prognosis 

a) Sacral sparing
b) Vibration at ankle 


63. Most common location of non-traumatic OCD in the talus


a) Posteromdial
b) Posterolateral
c) Anteromedial
d) Anterolateral

64. Sever disease most commonly associated with 



a) Calcaneus 

b) Talus
c) Radius
d) Capitulum 


65. Stick runner athlete presented with wrist snapping pain during wrist motion due to ECU.
What kind of motion that induces the pain:

a) flexion

b) extension
c) pronation
d) supination

66. Madelung deformity involves


a) Associated with Rickets

b) Bilateral in 90%

c) Non-progressive
d) Loss of supination
67. Pt with simple mid shaft tibia fracture fixed by plate. What is adequate description of
the blood supply
a) Neutrient blood supply the inner 2/3
b) immature bone flow is from periosteal to nutrient
c) Periosteal blood flow has high pressure

d) Neutrient blood supply has low pressure

68. Positive Trendelenburg gait, what nerve can cause it?


a) Femoral

b) Obturator

c) Superior gluteal
d) Inferior gluteal

69. Pt with Rheumatoid arthritis, what is commonly seen?


a) Periarticular erosion

70. Posterior Acetabulum wall fracture, which approach you will use to fix it?
a) Ilioinguinal
b) Kocher- Langenbeck
c) Watson-Jones
d) Extensile Stoppa approach

71. Patient with posterior acetabular wall fracture, what is the best view
a) Inlet

b) Outlet

c) Iliac
d) Obturator

72. Stab wound at axilla with radial nerve complete cut


a) Thumb abduction preserved

b) pronation preserved
c) Elbow extension preserved
d) thumb extension preserved

73. Osteopenia apparent in X-ray


a) 30-35
b) 40-45

74. Which of the following is a radiological feature in neuropathic knee arthropathy


a) Fragmentation and subluxation
b) Subchondrol sclerosis, periosteal cyst

75. Type 1 osteoporosis with vertebral fracture, the main cause of the fracture - Decrease
water will affect axial load

a) Cortex will be affected with loss of estrogen

b) Trabeculae .....
76. Femur fracture treated with plate and screws
a) Inner diameter indicates the pull-out strength
b) Outer diameter indicate tension

77. Male patient with 3 parts proximal humerus fracture. Comparing to deltopectoral
approach what is the structure under risk in Deltoid splitting approach:

a) Axillary nerve
b) anterior circumflex humeral artery
c) Posterior circumflex humeral artery
d) Musculocutaneous nerve

78. What will be found when a nerve is completely transacted


a) Distal myelin phagocytosed
b) Distal Schwan die

79. In osteoarthritis
a) Increase articular cartilage stiffness
b) Decrease Chondrocyte size

c) Increase chondrocyte number

d) Increase total Proteoglycan concentration

80. Above knee amputation the metabolic demand for the patient to walk:
a) 25-30

b) 30-40

c) 40-50
d) 50-65

81. what is the effect of nicotine in wound healing in heavy smoker pt. :
a) Decrease the time of healing

b) Increase the fracture callus strength

c) Increase the pseudoarthrosis in spine fusion

82. Pt with Complex Regional Pain Syndrome, what is the best treatment modality?
a) Hydrotherapy

b) Electrical stimulation

c) Ultrasonography
d) Gentle physiotherapy

83. 60-year-old male with Proximal femur lytic lesion presented with fracture
CT done no
other lesions were detected .. What is the next step in management?
a) open biopsy
b) Antegrade nail

c) Retrograde nail
84. MM most diagnostic test
a) Bone marrow aspiration
b) CBC

c) Lateral skull x-ray

d) CT scan of the chest

85. Which of the following is best diagnosed with Skeletal survey rather than bone scan
a) MM
b) Osteogenic Sarcoma
c) Ewing sarcoma

86. 12 years old c/o Leg pain X-ray showed periosteal reaction biopsy showed Small round
blue cells .. What is the Dx ?

a) Osteosarcoma

b) Ewing sarcoma
c) Giant cell tumor

87. what is common tumor metastasis to regional lymph node:


a) Liposarcoma

b) Leiomyosarcoma

c) Epithelioid Sarcoma
d) fibromyxosarcoma


88. Osteoid osteoma with night pain


a) NSAID

89. The cause of pain in osteoid osteoma:


a) Osteoclast activity

b) Prostaglandin release
c) integrity of structural bone

90. Which procedure has the least tumor recurrence with Distal femur osteosarcoma
a) AKA with wide resection
b) Megaprosthesis of tibia and femur
c) Revision TKR with

91. Sarcoma on the posterior aspect pf the thigh, what should be avoided when taking
biopsy
a) Open biopsy

b) Core biopsy
c) Transverse incision
d) longitudinal incision
92. What’s most common cause of epiphyseal tumor after GCT:
a) Osteoblastoma

b) Chondroblastoma
c) Aneurismal bone cyst
d) Conventional Chondrosarcoma

93. Femur Lytic lesion, what is mostly correlated with risk of fracture?
a) Age
b) Gender

c) Location of the lesion
d) Histologic type

94. The most common clinical manifestation of hemolytic transfusion reaction is:
a) Flank pain

b) Jaundice

c) Oliguria
d) A shaking chill

95. what is leading cause of death following blood transfusion:


a) ABO hemolytic transfusion reaction

b) Transfusion related acute lung injury
c) 3. Bacterial Contamination of platelet
d) 4. Iatrogenic hepatitis C infection

96. After needle prick injury what is the percentage of HIV :


a) < 0.5
b) 1-4

c) 5-10
d) 10-15

97. Common organism in necrotizing fasciitis


a) Group A streptococcus

98. what type of bacterial sepsis cause thrombocytopenia and hemorrhagic disorder
a) Tuberculosis

b) Encapsulated bacteria

c) Gram positive
d) Gram negative

99. Which of the following is a dominant cytokine in the pathogenesis of systemic


inflammatory response syndrome:

a) IL2
b) IL5
c) IL6
d) IL7
100. Cause of cardiogenic shock –
a) Air embolism
b) Hemothorax
c) PE
d) Excessive fluid resuscitation

101. RTA victim with excessive fluid resuscitation, will result in:
a) Metabolic acidosis
b) Metabolic alkalosis
c) Respiratory acidosis

102. 27y female involved in a MVA she brought to ER unconscious, intubated, diminished
breath sound over the Right chest, BP 80/10, HR 120 and distended abdomen, what is the
next step in management?

a) Laparotomy
b) Needle decompression
c) Chest tube
d) O-ve blood transfusion

103. Trauma pt , what is the best indicator for adequate resuscitation after 8 hrs:
a) Hypothermia
b) Lactic acid < 2
c) Alb 35
d) Urine output 0.3ml/kg/hr

104. Immunologic rejection is mediated by the recipient's:


a) Eosinophils

b) Lymphocytes
c) Neutrophils
d) Plasma cells

105. 70 kg patient stabbed in the abdomen & was brought to ER He was hypotensive,
markedly tachycardia and confused.
What is the expected blood loss?
a) 10-15%
b) 15-30%
c) 30-35%
d) 50-55%

106. When taken concomitantly with which of the following, erythromycin can cause an
increase in the blood level of
a) digoxin.
b) sucralfate.
c) clindamycin.

107. Patient with Gustillo 2 open fracture, allergic to penicillin and cephalosporin, which
antibiotics he should receive?
a) Clindamycin
108. Steroids impair wound healing by:
a) Decreasing angiogenesis and macrophage migration
b) Decreasing platelet plug integrity
c) Increasing release of lysosomal

109. Regarding ESR, CRP which one is true


a) CRP elevated 12hr post OP
b) ESR decrease after 3 weeks
c) CRP decrease after 2 weeks

110. What is the type of distribution among these 13, 13, 12, 14, 14, 14, 5, 6

a) Non-modal

b) Bimodal
c) Unimodal
d) Multimodal

111. 14 y girl presented with her mother diagnosed with appendicitis and her father is away,
about the consent

a) Wait for father

b) Explain to the girl and take consent from mother

112. A clinical trial for new medication the reduce the risk of suicide in severely depressed
patients. Consent was taken from Depressed patient who are not controlled on their
medications

a) Acceptable since voluntary consent taken
b) Unacceptable since not approved meds
c) Not acceptable as the participant lack .....

113. Study conducted on hospital 100 Interns to evaluate their weight, the mean was found
to be 75+-10 how many of them will have a weight range from 65-85 kg ?

a) 30-40%

b) 60-70%
c) 80-90%
d) 95%

114. A study was done on new born babies of educated and non-educated mothers and
found that babies of educated mothers have weight 3200 +-150 and non-educated is 2800 +-
200 what is the type of the study is this?
a) T test
b) Chi square
c) Cohort

d) Case control
115. A new skin cancer lesion was found to be in employee of a plastic factory. A study was
done to see if there is a relation between their job and the disease what is the type of the
study is this?
a) Cross sectional
b) Cohort

c) Case control

116. Someone from pharmacy company came to our clinic & explain to you about their drug
& it is already used in another country: 

a) Take sample for trial
b) Prescribe the drug since it is used in other countries
c) The company should provide efficiency of the medication

117. One of the following is a Medical Error definition:


a) Mistaken Procedure or prescription
b) Unintentional action taken by doctor
c) Negligence

d) Management plan not completed as intended

118. Which of the following would be the main reason to consider research participants
vulnerable?
a) Poorer than other participants
b) Unable to protect their interests
c) Persons with emotional distress
d) Mentally disabled or handicapped

119. Question about type of study .. Cohort

120. Question about research consent


Saudi Orthopedic Part One Exam 2018
1. Intrinsic muscles of the foot which layers:
a) 2nd and 4th
b) 1st and 3rd
c) 1st and 4th
d) 2nd and 3rd

2. Case of neuromuscular scoliosis was done with fusion of posterior approach, post op
days 7 he presented to ER with abdominal pain, distension, dehydration and vomiting.
What is the most likely diagnosis?
a) Stress ulcer.
b) Superior mesenteric artery syndrome.
c) Postopertive ileus.
d) Retroperioteneal hematoma.

3. What is the preferred position for hip arthrodesis?


a) 15 degrees of hip flexion, 0 degrees of abduction, 15 degrees of external rotation.
b) 20 degrees of hip flexion, 0 degrees of abduction, 5 degrees of external rotation.
c) 15 degrees of hip flexion, 0 degrees of abduction,5 degrees of external rotation.
d) 20 degrees of hip flexion, 0 degrees of abduction, 20 degrees of external rotation.

4. The anterior portal of a hip arthroscopy places which of the following structure at
greatest risk for injury?
a) Ascending branch of the lateral circumflex femoral artery.
b) Branch of the medial circumflex femoral artery.
c) Femoral nerve.
d) Lateral femoral cutaneous nerve.

5. Pincer impingement is associated with … :


a) Femoral anteversion.
b) Femoral retroversion.
c) Acetabular anteverion.
d) Acetabular retroversion.

6. Which of the following is the most common complication following a hip revision with
isolated polyethylene (PE) exchange for osteolysis?
a) Nerve injury.
b) Heterotopic ossification.
c) Dislocation.
d) Infection.

7. Which of the following factors improves the performance of cemented femoral stems in
THA?
a) Stiffer stem materials.
b) Calcar collar contact.
c) Decreased thickness of the non-cemented mantle.
d) Decreased thickness of the cemented mantle.
8. During a revision of a failed total hip femoral component for osteolysis, the new implant
should bypass the most distal cortical defect by a minimum of how many cortical
diameters?
a) 1
b) 2
c) 3
d) 4

9. During THA, profuse bleeding is noted following predrilling for placement of an


acetabular component screw in the antrosuperior quadrant. The drill most likely
penetrated which structure:
a) External iliac artery.
b) Internal iliac artery.
c) Femoral artery.
d) Obturator artery.

10.Indication of TSA is … :
a) Shoulder OA with intact rotator cuff muscle.

11.A 25-year-old male injured his left knee in a motor cycle accident. During examination he
is noted to have a positive reverse pivot shift test and a negative posterior drawer. What
other examination finding is this patient expected to have?
a) Positive anterior drawer test
b) Increased opening to valgus stress at 30 degrees of knee flexion
c) Positive apprehension sign with lateral patellar translation
d) A 10-degree increased external tibial rotation at 30 degrees of knee flexion
e) A 10-degree increased external tibial rotation at 90 degrees of knee flexion

• A positive "Dial Test" at 30 degrees of knee flexion would be expected in this


patient with a posterolateral corner (PLC) injury. The test is positive when there is
greater than a 10-15 degree difference compared to the uninjured side. A positive
dial test at 30 degrees alone indicates an isolated PLC injury whereas a positive
dial test at 30 and 90 degrees indicates a combined PLC and PCL injury. A PLC
injury should be suspected based on the given exam findings of a positive reverse
pivot shift and a negative posterior drawer. This indicates an injury to the PLC but
not the posterior cruciate ligament (PCL).

12.In type II acromioclavicular separation, which is true :


a) CC is intact and AC is intact.
b) CC is torn and AC is intact.
c) CC is intact and AC is torn.
d) CC is torn and AC is torn.

13.Mechanism of aspirin
a) Thromboxane a2 inhibition.
b) Reverse Cyclooxygenase inhibition.
14.A woman who is in the 20th week of her pregnancy seeks an orthopedic consultation
after undergoing an ultrasound. The findings reveal that the fetus has bilateral clubfeet
and both femurs measure less than two standard deviations below normal. What is the
most likely diagnosis?
a) Myelomeningocele
b) Bilateral proximal focal femoral deficiency
c) Diastrophic dysplasia
d) Achondroplasia
e) Spondylometaphyseal dysplasia

15.component of carpal tunnel?


a) Median nerve with 9 flexors.
b) Median nerve with 8 flexors.

16.Madelung deformity?
a) Wrist.
b) Elbow.
c) Hip.
d) Knee.

17.Internal rotation and valgus stress of the tibia, what is primary restraint?
a) Posterior medial corner.
b) MPFL.
c) Posterior oblique ligament.

• POL is the Primary stabilizer against internal rotation and valgus between 0 and 30
degrees of knee flexion 


18.Compared with patients who have other conditions, patients with sickle cell disease
experience what complication more frequently following total hip arthroplasty?
a) Heterotopic ossification
b) Dislocation
c) Loosening
d) Fracture

19.Case of club foot, ponseti method of treatment correction start with:


a) Midfoot cavus.
b) Forefoot adduction.
c) Hindfoot Varus.
d) Hindfoot equines.

20.elderly patient with distal radius fracture , underwent ORIF , developed post op
numbness over palmar aspect 1st ,2nd 3rd finger (CTS) what is best next:
a) CTS release.
21.Saphenous vein injury common on which of the following players
a) Football.
b) Surfers.
c) Rowers.
d) Golfer.

• Thrombosis of the Greater Saphenous Vein in a Collegiate Football Place Kicker:


The greater saphenous vein is vulnerable in its superficial course overlying the
distal tibia in the zone of impact of a kicker's foot Minor repetitive trauma or a
single traumatic event to this area may subject the vein to intimai trauma or other
irregularities that may lead to vein thrombosis.

22. Jefferson burst fracture?


a) C1 body.
b) C1 ring.
c) C2 body.
d) C3 body.

23.Trauma pt. Hypotensive, tachycardia, intubated, diminished right chest sound , abdomen
distended, fast -ve, what in the best next step?
a) Chest tube.
b) DPL.
c) Needle.

24.A 25-year-old man with multiple injuries has an injury severity score of 40. His risk of
mortality from his injuries is
b) 0%.
c) 20%.
d) 50%.
e) 80%.
f) 100%.

25.Which of the following cells has parathyroid hormone receptor:


a) Osteoblast.
b) Osteoclast.
c) Osteocyte.

26.What is the anatomic relation between popliteal tendon and LCL


a) Anterior distal.
b) Anterior proximal.
c) Posterior distal.
d) Posterior proximal.

• Popliteus tendon Femoral insertion is distal, anterior, and deep to the LCL 

27.What is the best surgical indication for meniscal tear?
a) Red white zone.
b) Horizontal.
c) 1-4 cm tear.

• Meniscal repair Indications:


✓ Tear between 1 and 4 cm
✓ Vertical tear
✓ Red-Red tear
✓ Meniscal root tear
✓ Age younger than 40 years 


28.Which pulley should be preserved in repair of flexor tendon :


a) A1 and A2.
b) A1 and A3.
c) A2 and A4.
d) A3 and A4.

29.What is the most common complication of open SH injury type I and II of distal phalanx:
(Pre test)
a) Non-union.
b) Infection.
c) Growth arrest.
d) Deformity

30.16-year boy had closed head injury and loss of consciousness for 10 mints while playing
football what is the correct recommendation? (Pre-test 352)
a) Restriction from practicing and competition for 3 days.
b) Restriction from practicing and competition for 1 month.
c) Restriction from practicing and competition for 6 months
d) Immediately return to practicing

31. A 54-year-old female presents with a hand deformity. A surgical procedure is being
considered that relocates the lateral bands dorsally to counteract the pathophysiology of
the deformity. Which of the following deformities does this patient most likely have?
a) Boutonniere Deformity.
b) Mallet Finger Deformity.
c) Jersey Finger Deformity.
d) Swan Neck Deformity.

• (Orthobullet) An acute boutonnière deformity results from central slip disruption


and volar subluxation of the lateral bands, resulting in DIP hyperextension

32.What abnormality will be found in a patient with class III Hypovolemic Shock?
a) Decreased Blood Pressure.
b) Confusion and lethargy.
c) Heart Rate Less 100 p/min.

33. You are working as an ER physician and your shift ended at 8:00 AM the physician from
the next shift has not arrived yet, a patient entered ER at 8:15 AM what is the
appropriate action to do:
a) Assess the patient and stabilize him/her.
b) Leave ER at 8 So the physician will be blamed.
c) Let the patient wait in the bed till the physician arrive.

34. The Evans lateral calcaneal lengthening osteotomy is the surgical procedure most
appropriate for which pediatric foot deformity?
a) Flexible pes planovalgus.
b) Cavus Foot.
c) Talipes Equinovarus.
d) Juvenile Hallux Valgus.

35. Which of the following procedure if done alone will worsen crouched gait in patient with
spastic cerebral palsy :
a) Lengthening of Heel cord.
b) Tenotomy of iliopsoas from the insertion site at lesser trochanter.
c) Hamstring lengthening.

36. What is the most important step in transfemoral amputation?


a) Adductors myodesis.

37. Most predictive factor of postoperative anemia:


a) Preoperative Hg Level.
b) Age.
c) Weight.

38. A 7-year-old girl undergoes open reduction internal fixation of a displaced humeral
lateral condyle fracture. Dissection around which portion of the fracture fragment should
be avoided to protect its blood supply:
a) Medial.
b) Lateral.
c) Superior.
d) Posterior.

39.A 60-year-old man found to have aggressive lytic lesion what is the most common cause
of this finding?
a) MM.
b) Lymphoma.
c) Metastatic Bone disease.
40.A question about feature of dysplastic spondylolisthesis:
a) Trapezoid L5 - Round S1.

• Dysplastic types include dome-shaped or significantly inclined sacrum (>30 degrees


beyond vertical position), trapezoid-shaped L5, sagittally shaped or dysplastic facets
of S1.

41. which of the following is microscopic features of gout crystals :
a) Green under polarized light microscope.
b) Monosodium urate.

42. Which of the following tests is required for a standard work-up of Ewing's sarcoma that
is not routinely obtained for staging of osteosarcoma? (Orthobullet)
a) MRI.
b) CT scan.
c) Bone scan.
d) Bone marrow biopsy.

43.which of the following best describe nerve root for SLR test and reverse SLR
a) L4,L5
b) L5, S1
c) L2, L3,

44. 46 year old lady with middle finger flexion deformity, palpable lesion in the palm,
excision of the lesion will show:
a) Inflammatory granuloma.
b) Fibromatosis.
c) Giant cell tumor of tendon sheath.

• Palmar fibromatosis (Dupuytren’s contracture)

45.Meralgia paresthetica:
a) LCFN.
b) Sural.

46. Pt underwent THA, instead of giving him an appropriate prophylactic dose of warfarin
the surgeon gave him LMWH, which complication most likely to happen:
a) Surgical site hematoma.
b) DVT in thigh.
c) DVT in leg.

47.Risk factor of adhesive capsulitis:


a) Neck surgery.
b) Thyroid disease.
48. In a patient with -2.5mm of ulnar variance, which of the following statements best
describes the distribution of compressive load across the wrist? (Orthobullet)
a) Approximately 50% of the wrist load is accepted by distal radius and 50% is accepted by
the distal ulna.
b) Approximately 80% of the wrist load is accepted by the distal radius and 20% is
accepted by the distal ulna.
c) Approximately 80% of the wrist load is accepted by the distal ulna and 20% is accepted
by the distal radius.
d) Approximately 95% of the wrist load is accepted by the distal radius and 5% is
accepted by the distal ulna.
e) Approximately 60% of the wrist load is accepted by the distal radius and 40% is
accepted by the distal ulna.

• Ulnar variance describes the cranio-caudal position of the distal ulna in relation to
the distal radius at the wrist. In neutral ulnar variance, 80% of the compressive load
across the wrist is accepted by the distal radius, and 20% is accepted by the distal
ulna. With -2.5mm of ulnar variance (negative ulnar variance), approximately 5% of
the wrist load is accepted by the distal ulna. With +2.5mm of ulnar variance (positive
ulnar variance), approximately 40% of the wrist load is accepted by the distal ulna.

49.Most common meniscal injury:


a) Anterior horizontal.
b) Posterior horizontal.
c) Bucket handle.

• The posterior horn is also the most common area for degenerative tears of
the meniscus

50.12 year old boy, found to have complete mid substance ACL rupture and MCL tibial part
avulsion, the knee is grossly unstable with valgus stress and anterior posterior applied
force, the most appropriate management: (Pre test)
a) MCL repair and ACL rehab.
b) Surgical repair of MCL and intraarticular ACL recon.
c) Immobilization and early ROM.

51.Which is correct about cartilage injury?


a) Autograft for 15cm defect.
b) Injury under tidemark has less potential to heal.
c) Early mobilization not helpful for healing.

• Cartilage is anisotropic, most of the water is located in the superficial layers, it only
heals if the injury does pass through the tidemark, type X collagen is found in
calcified cartilage and is thought to be involved in mineralization.
• < 4 cm2 = microfracture or osteochondral autograft
52.MPFL injury, which of the following best treated with MPFL reconstruction:
a) TTGT 12
b) TTGT 13 with trochlea dysplasia.

53.Load of rotation of spine:


a) 30% disc, 30% facet joint, 30% posterior ligaments.
b) 40% disc, 20% facet joint, 40% posterior ligaments
c) 10% disc, 40% facet joint, 30% posterior ligaments
d) 50% disc, 30% facet joint, 20% posterior ligaments

• Torsional load resistance in the lumbar spine:


✓ Facets contribute 40%
✓ Disc contributes 40%
✓ Ligamentous structures contribute 20%

54.14-year-old patient who has homocystinuria and is Risser 3 is referred for surgical
treatment of scoliosis. In addition to the usual risks associated with posterior spinal
fusion, the family should be advised that the patient's underlying condition significantly
increases the perioperative risk of
a) spinal pseudarthrosis.
b) spinal cord traction injury with paralysis.
c) arterial and venous thromboses.
d) superior mesenteric artery syndrome.
e) crankshaft phenomenon.

55.Child with myelomeningocele lumber region presented with lower limb weakness and
scoliosis T10-L2, the most likely diagnosis:
a) Tethering of the spine.
b) Muscular scoliosis.

56.radiograph of a 12-year-old boy who has had an insidious onset of pain in the right hip
for the past 6 weeks shows diffuse narrowing of the joint space. Examination reveals that
he is afebrile, and the range of motion of the hip is less than 50% of normal in all planes.
Laboratory studies show an erythrocyte sedimentation rate of 21 mm/hr and a VVBC of I
1,000/mm3. What is the most likely diagnosis? (OITE 98)
a) Sickle cell crisis
b) Idiopathic chondrolysis
c) Hemophilic arthropathy
d) Osteoid osteoma of the femoral neck
e) Legg-Calve-Perthes disease

• Idiopathic chondrolysis is a rapidly progressive chondrolysis occurring during


adolescence without demonstrable etiology.
• painful ache, insidious onset, associated with progressive joint stiffness and limp.
• global restriction of motion in all planes
• X-rays: narrowing of joint space to
57.Anterior approach to the hip the surgeon faced bleeding which structure was injured:
a) Ascending branch from the LFCA
b) Descending branch of LFCA
c) Femoral artery
d) Pudendal artery

58.After ankle triple arthrodesis which causes psudoarthrosis:


a) Talocalcaneal.
b) Talonavicular.
c) Calcanocuboid.

59.Wormian bones seen in:


a) Paget disease.
b) Osteogenesis imperfecta.

60.Spastic diplegia pt did MRI, which of the following most likely to be seen:
a) Microcephaly.
b) Peri ventricular leukomalacia.
c) Cyst.

61.Waddling gait cause:


a) Gluteus weakness.
b) Adductor weakness.

62.SLR and femoral stretch test which level:


a) L4-L5 / L5-S1 and L2-L3 / L3-L4

63.Most common anerobic bacteria in axilla:


a) P. acnes.

64. Effect of patellectomy:


a) 60% reduction in extensor efficiency.

• Patellectomy
✓ Length of the moment arm is decreased by width of patella: 30% reduction
✓ Power of extension is decreased by 30%.

65.Case of low grade soft tissue sarcoma. Surgeon found +ve biopsy at biopsy margin. What
to do?
a) Chemotherapy.
b) Radiotherapy.
c) Re-excision of tumor bed.

66.Biopsy taken from fracture site after 2 weeks. It will show:


a) Chondrocyte in proliferative.
b) Chondrocyte in hypertrophic.
67.Examination of a 5-year-old child who has fibular hemimelia reveals the foot has two
rays and is stiff in equinus and valgus. The level of the foot is just proximal to the mid-
shaft of the contralateral tibia, and the knee has full active flexion and extension, but
slight valgus, Treatment should include
a) amputation through the mid-shaft of the tibia.
b) correction of the foot deformity and lengthening with a ring fixator.
c) knee disarticulation.
d) ankle disarticulation.
e) ankle disarticulation and contralateral epiphyseodesis of the proximal tibia

68.Case of ulnar hypertrophic nonunion management include?


a) ORIF with bone grafting.
b) ORIF without bone grafting.
c) CRIF.
d) Observation.

69.Immediate source of energy in muscle:


a) Glycolytic.
b) Oxidative (Krebs cycle).
c) ATP

70.If a 75-year-old man with no prior history of back or leg pain were to undergo an MRI of
the lumbar spine, what is the chance that the study would demonstrate disc
degeneration and/or bulging?
a) 20%
b) 35%
c) 50%
d) 70%
e) 90%

71.Which of the following patients with Legg-Calve-Perthes disease would have the most
guarded prognosis?
a) A 4-year-old girl with partial head involvement
b) A 4-year-old boy with partial head involvement
c) A 4-year-old girl with collapse of the lateral pillar
d) A 6-year-old boy with total head involvement
e) A 9-year-old boy with collapse of the lateral pillar

Collected by Saud almaslmani

You might also like